Author Archives: Hung Nguyen

PHÉP NHÂN VÀ PHÉP CHIA ĐA THỨC – P.2

CÁC HẰNG ĐẲNG THỨC ĐÁNG NHỚ

 

Thực hiện phép nhân đa thức, ta được các hằng đẳng thức sau :

1. $(a+b)^2=a^2+2 a b+b^2$.

2. $(a-b)^2=a^2-2 a b+b^2$.

3. $(a+b)(a-b)=a^2-b^2$.

4. $(a+b)^3=a^3+3 a^2 b+3 a b^2+b^3$

$(a+b)^3=a^3+b^3+3 a b(a+b) \text {. }$

5. $(a-b)^3=a^3-3 a^2 b+3 a b^2-b^3$

$(a-b)^3=a^3-b^3-3 a b(a-b)$

6. $(a+b)\left(a^2-a b+b^2\right)=a^3+b^3$

7. $(a-b)\left(a^2+a b+b^2\right)=a^3-b^3$.

Ta cũng có :

$(a+b+c)^2=a^2+b^2+c^2+2 a b+2 a c+2 b c .$

Tổng quát của các hằng đẳng thức 3 và 7 , ta có hằng đẳng thức :

8. $a^n-b^n=(a-b)\left(a^{n-1}+a^{n-2} b+a^{n-3} b^2+\ldots+a b^{n-2}+b^{n-1}\right)$

với mọi số nguyên dương $\mathrm{n}$.

Tổng quát của hằng đẳng thức 6 , ta có hằng đẳng thức :

9. $a^n+b^n=(a+b)\left(a^{n-1}-a^{n-2} b+a^{n-3} b^2-\ldots-a b^{n-2}+b^{n-1}\right)$

với mọi số lẻ n.

Tổng quát của các hằng đẳng thức $1,2,4,5$, ta có công thức Niu-tơn (xem chuyên đề Tính chia hết đối với số nguyên).

Ví dụ 1. Chứng minh rằng số 3599 viết được dưới dạng tích của hai số tự nhiên khác 1 .

Giải : $\quad 3599=3600-1=60^2-1=(60+1)(60-1)=61.59$.

Ví dụ 2. Chứng minh rằng biểu thức sau viết được dưới dạng tổng các bình phương của hai biểu thức :

$x^2+2(x+1)^2+3(x+2)^2+4(x+3)^2$

Giải: $\mathrm{x}^2+2(\mathrm{x}+1)^2+3(\mathrm{x}+2)^2+4(\mathrm{x}+3)^2=$

$=x^2+2\left(x^2+2 x+1\right)+3\left(x^2+4 x+4\right)+4\left(x^2+6 x+9\right) $

$=x^2+2 x^2+4 x+2+3 x^2+12 x+12+4 x^2+24 x+36 $

$=10 x^2+40 x+50 $

$=\left(x^2+10 x+25\right)+\left(9 x^2+30 x+25\right) $

$=(x+5)^2+(3 x+5)^2$

Ví dụ 3. Cho

$x+y+z=0 $

$4x y+y z+z x=0$

Chứng minh rằng $\mathrm{x}=\mathrm{y}=\mathrm{z}$.

Giải : Ta có $(\mathrm{x}+\mathrm{y}+\mathrm{z})^2=\mathrm{x}^2+\mathrm{y}^2+\mathrm{z}^2+2(\mathrm{xy}+\mathrm{yz}+\mathrm{zx})$.

Suy ra

$0=x^2+y^2+z^2+2.0$

hay

$\text { Vậy } x=y=z(=0) \text {. }$

Ví dụ 4 :

a) Tính $A=-1^2+2^2-3^2+4^2-\ldots-99^2+100^2$.

b) Tính $\mathrm{A}=-1^2+2^2-3^2+4^2-\ldots+(-1)^{\mathrm{n}} \cdot \mathrm{n}^2$.

Giải: a) $\mathrm{A}=\left(2^2-1^2\right)+\left(4^2-3^2\right)+\ldots+\left(100^2-99^2\right)$

$=(2-1)(1+2)+(4-3)(3+4)+\ldots+(100-99)(99+100) $

$=1+2+3+4+\ldots+99+100 $

$=\frac{100.101}{2}=5050 .$

b) Xét hai trường hợp :

Nếu n chẵn thì $\mathrm{A}=\left(2^2-1^2\right)+\left(4^2-3^2\right)+\ldots+\left[\mathrm{n}^2-(\mathrm{n}-1)^2\right]$

$=1+2+3+4+\ldots+(n-1)+n$

$=\frac{\mathrm{n}(\mathrm{n}+1)}{2} \text {. }$

Nếu n lẻ thì $\mathrm{A}=\left(2^2-1^2\right)+\left(4^2-3^2\right)+\ldots+\left[(\mathrm{n}-1)^2-(\mathrm{n}-2)^2\right]-\mathrm{n}^2$

$=1+2+3+4+\ldots+(n-1)-n^2 $

$=\frac{n(n-1)}{2}-n^2=-\frac{n(n+1)}{2}$

Chú ý : Hai kết quả trên có thể viết chung trong một công thức

$(-1)^{\mathrm{n}} \cdot \frac{\mathrm{n}(\mathrm{n}+1)}{2}$

Ví dụ 5. Cho

$x+y=a+b\quad(1)$

$x^2+y^2=a^2+b^2\quad(2)$

Chứng minh rằng $x^3+y^3=a^3+b^3$.

Giải : Ta có : $\quad \mathrm{x}^3+\mathrm{y}^3=(\mathrm{x}+\mathrm{y})\left(\mathrm{x}^2-\mathrm{xy}+\mathrm{y}^2\right)\quad(3)$.

Từ (1) suy ra : $\quad(x+y)^2=(a+b)^2$,

tức là $\quad x^2+2 x y+y^2=a^2+2 a b+b^2$.

Do $x^2+y^2=a^2+b^2$ nên $2 x y=2 a b$, suy ra $x y=a b\quad(4)$

Thay các kết quả (1), (2), (4) vào (3), ta được

$x^3+y^3=(x+y)\left(x^2+y^2-x y\right)=(a+b)\left(a^2+b^2-a b\right)=a^3+b^3 .$

Ví dụ 6. Cho $a+b=m, a-b=n$. Tính $a b$ và $a^3-b^3$ theo $m$ và $n$.

Giải :

Cách 1. Từ $\mathrm{a}+\mathrm{b}=\mathrm{m}, \mathrm{a}-\mathrm{b}=\mathrm{n}$, ta tính được $\mathrm{b}=\frac{\mathrm{m}-\mathrm{n}}{2}, \mathrm{a}=\frac{\mathrm{m}+\mathrm{n}}{2}$.

Do đó $\quad \mathrm{ab}=\frac{\mathrm{m}+\mathrm{n}}{2} \cdot \frac{\mathrm{m}-\mathrm{n}}{2}=\frac{\mathrm{m}^2-\mathrm{n}^2}{4} ;$

$a^3-b^3=\left(\frac{m+n}{2}\right)^3-\left(\frac{m-n}{2}\right)^3=\frac{(m+n)^3-(m-n)^3}{8}$

Rút gọn biểu thức trên, ta được $\frac{3 \mathrm{~m}^2 \mathrm{n}+\mathrm{n}^3}{4}$.

Cách 2. Ta có

$4 a b =(a+b)^2-(a-b)^2=m^2-n^2 \text { nên } a b=\frac{m^2-n^2}{4} . $

$\text { Ta có } a^3-b^3 =(a-b)\left(a^2+a b+b^2\right)=(a-b)\left[(a+b)^2-a b\right] $

$=n\left(m^2-\frac{m^2-n^2}{4}\right)=\frac{n\left(3 m^2+n^2\right)}{4}=\frac{3 m^2 n+n^3}{4} .$

BÀI TẬP

16. Tính giá trị của các biểu thức :

a) $\frac{63^2-47^2}{215^2-105^2}$

b) $\frac{437^2-363^2}{537^2-463^2}$

17. So sánh $\mathrm{A}=26^2-24^2$ và $\mathrm{B}=27^2-25^2$.

18. Tìm $\mathrm{x}$, biết :

$4(x+1)^2+(2 x-1)^2-8(x-1)(x+1)=11$

19. Rút gọn các biểu thức :

a) $2 x(2 x-1)^2-3 x(x+3)(x-3)-4 x(x+1)^2$;

b) $(a-b+c)^2-(b-c)^2+2 a b-2 a c$;

c) $(3 x+1)^2-2(3 x+1)(3 x+5)+(3 x+5)^2$;

d) $(3+1)\left(3^2+1\right)\left(3^4+1\right)\left(3^8+1\right)\left(3^{16}+1\right)\left(3^{32}+1\right)$;

e) $(a+b-c)^2+(a-b+c)^2-2(b-c)^2$

g) $(a+b+c)^2+(a-b-c)^2+(b-c-a)^2+(c-a-b)^2$;

h) $(a+b+c+d)^2+(a+b-c-d)^2+(a+c-b-d)^2+(a+d-b-c)^2$.

20. Cho $x+y=3$. Tính giá trị của biểu thức

$A=x^2+2 x y+y^2-4 x-4 y+1 $

21. Cho $\mathrm{a}^2+\mathrm{b}^2+\mathrm{c}^2=\mathrm{m}$. Tính giá trị của biểu thức sau theo $\mathrm{m}$ :

$A=(2 a+2 b-c)^2+(2 b+2 c-a)^2+(2 c+2 a-b)^2 .$

22. Hãy viết các số sau đây dưới dạng tích của hai số tự nhiên khác 1 :

a) $899$

b) $9991$

23. Chứng minh rằng hiệu sau đây là một số gồm toàn các chữ số như nhau :

$7778^2-2223^2$

24. Chứng minh các hằng đẳng thức :

a) $(a+b+c)^2+a^2+b^2+c^2=(a+b)^2+(b+c)^2+(c+a)^2$

b) $x^4+y^4+(x+y)^4=2\left(x^2+x y+y^2\right)^2$

25. Cho $\mathrm{a}^2-\mathrm{b}^2=4 \mathrm{c}^2$. Chứng minh hằng đẳng thức

$(5 a-3 b+8 c)(5 a-3 b-8 c)=(3 a-5 b)^2$

26. Chứng minh rằng nếu $\left(a^2+b^2\right)\left(x^2+y^2\right)=(a x+b y)^2$ với $x, y$ khác 0 thì $\frac{\mathrm{a}}{\mathrm{x}}=\frac{\mathrm{b}}{\mathrm{y}}$

27. Chứng minh rằng nếu $\left(\mathrm{a}^2+\mathrm{b}^2+\mathrm{c}^2\right)\left(\mathrm{x}^2+\mathrm{y}^2+\mathrm{z}^2\right)=(\mathrm{ax}+\mathrm{by}+\mathrm{cz})^2$ với $x, y, z$ khác 0 thì $\frac{a}{x}=\frac{b}{y}=\frac{c}{z}$.

28. Cho $(a+b)^2=2\left(a^2+b^2\right)$. Chứng minh rằng $a=b$.

29. Chứng minh rằng $\mathrm{a}=\mathrm{b}=\mathrm{c}$ nếu có một trong các điều kiện sau :

a) $a^2+b^2+c^2=a b+b c+c a$

b) $(a+b+c)^2=3\left(a^2+b^2+c^2\right)$

c) $(a+b+c)^2=3(a b+b c+c a)$.

  1. Hãy viết các biểu thức sau dưới dạng tổng của ba bình phương :

a) $(a+b+c)^2+a^2+b^2+c^2$

b) $2(a-b)(c-b)+2(b-a)(c-a)+2(b-c)(a-c)$

31. Tính giá trị của biểu thức $\mathrm{a}^4+\mathrm{b}^4+\mathrm{c}^4$, biết rằng $\mathrm{a}+\mathrm{b}+\mathrm{c}=0$ và :

a) $a^2+b^2+c^2=2$;

b) $a^2+b^2+c^2=1$.

32. Cho $\mathrm{a}+\mathrm{b}+\mathrm{c}=0$. Chứng minh $\mathrm{a}^4+\mathrm{b}^4+\mathrm{c}^4$ bằng mỗi biểu thức :

a) $2\left(a^2 b^2+b^2 c^2+c^2 a^2\right)$;

b) $2(a b+b c+c a)^2$

c) $\frac{\left(a^2+b^2+c^2\right)^2}{2}$

33. Chứng minh rằng các biểu thức sau luôn luôn có giá trị dương với mọi giá trị của biến :

a) $9 x^2-6 x+2$

b) $\mathrm{x}^2+\mathrm{x}+1$

c) $2 x^2+2 x+1$.

34. Tìm giá trị nhỏ nhất của các biểu thức :

a) $A=x^2-3 x+5 ;$

b) $B=(2 x-1)^2+(x+2)^2$

35. Tìm giá trị lớn nhất của các biểu thức :

a) $A=4-x^2+2 x$

b) $B=4 x-x^2$

36. Chứng minh rằng :

a) Nếu $\mathrm{p}$ và $\mathrm{p}^2+8$ là các số nguyên tố thì $\mathrm{p}^2+2$ cũng là số nguyên tố.

b) Nếu $\mathrm{p}$ và $8 \mathrm{p}^2+1$ là các số nguyên tố thì $2 \mathrm{p}+1$ cũng là số nguyên tố.

37. Chứng minh rằng các số sau là hợp số :

a) 999991 ;

b) 1000027 .

38. Thực hiện phép tính :

a) $(x-2)^3-x(x+1)(x-1)+6 x(x-3)$

b) $(x-2)\left(x^2-2 x+4\right)(x+2)\left(x^2+2 x+4\right)$.

39. Tìm $x$, biết :

a) $(x-3)\left(x^2+3 x+9\right)+x(x+2)(2-x)=1$

b) $(x+1)^3-(x-1)^3-6(x-1)^2=-10$

40. Rút gọn các biểu thức :

a) $(a+b+c)^3-(b+c-a)^3-(a+c-b)^3-(a+b-c)^3$

b) $(a+b)^3+(b+c)^3+(c+a)^3-3(a+b)(b+c)(c+a)$

41. Chứng minh các hằng đẳng thức :

a) $(a+b+c)^3-a^3-b^3-c^3=3(a+b)(b+c)(c+a)$.

b) $a^3+b^3+c^3-3 a b c=(a+b+c)\left(a^2+b^2+c^2-a b-b c-c a\right)$.

42. Cho $a+b+c=0$. Chứng minh rằng $a^3+b^3+c^3=3 a b c$.

43. Cho $\mathrm{x}+\mathrm{y}=\mathrm{a}$ và $\mathrm{xy}=\mathrm{b}$. Tính giá trị của các biểu thức sau theo $\mathrm{a}$ và $\mathrm{b}$ :

a) $x^2+y^2$

b) $x^3+y^3$

c) $x^4+y^4$;

d) $x^5+y^5$.

44. a) Cho $x+y=1$. Tính giá trị của biểu thức $x^3+y^3+3 x y$.

b) Cho $\mathrm{x}-\mathrm{y}=1$. Tính giá trị của biểu thức $\mathrm{x}^3-\mathrm{y}^3-3 \mathrm{xy}$.

45. Cho $\mathrm{a}+\mathrm{b}=1$. Tính giá trị của biểu thức

$M=a^3+b^3+3 a b\left(a^2+b^2\right)+6 a^2 b^2(a+b)$

46. a) Cho $x+y=2$ và $x^2+y^2=10$. Tính giá trị của biểu thức $x^3+y^3$.

b) Cho $x+y=a$ và $x^2+y^2=b$. Tính $x^3+y^3$ theo a và $b$.

47. Chứng minh rằng :

a) Nếu số n’ là tổng của hai số chính phương thì 2 n cũng là tổng của hai số chính phương.

b) Nếu số $2 \mathrm{n}$ là tổng của hai số chính phương thì n cũng là tổng của hai số chính phương.

c) Nếu số $\mathrm{n}$ là tổng của hai số chính phương thì $\mathrm{n}^2$ cũng là tổng của hai số chính phương.

d) Nếu mỗi số m và $\mathrm{n}$ đều là tổng của hai số chính phương thì tích mn cũng là tổng của hai số chính phương.

48. Chứng minh rằng với mọi số tự nhiên $\mathrm{a}$, tồn tại số tự nhiên $\mathrm{b}$ sao cho $\mathrm{ab}+4$ là số chính phương.

49. Cho a là số gồm $2 \mathrm{n}$ chữ số $1, \mathrm{~b}$ là số gồm $\mathrm{n}+1$ chữ số $1, \mathrm{c}$ là số gồm $\mathrm{n}$ chữ số 6. Chứng minh rằng $\mathrm{a}+\mathrm{b}+\mathrm{c}+8$ là số chính phương.

50. Chứng minh rằng biểu thức sau không là lập phương của một số tự nhiên :

$10^{150}+5.10^{50}+1 .$

51. Chứng minh rằng tích ba số nguyền dương liên tiếp không là lập phương của một số tự nhiên.

52. Chia 27 quả cân có khối lượng $10,20,30, \ldots, 270$ gam thành ba nhóm có khối lượng bằng nhau.

53*. Chia 18 quả cân có khối lượng $1^2, 2^2, 3^2, \ldots, 18^2$ gam thành ba nhóm có khối lượng bằng nhau.

54*. Chia 27 quả cân có khối lượng $1^2, 2^2, 3^2, \ldots, 27^2$ gam thành ba nhóm có khối lượng bằng nhau.

PHÉP NHÂN VÀ PHÉP CHIA ĐA THỨC – P.1

NHÂN ĐA THỨC

 

Ví dụ 1. Tính giá trị của biểu thức

$A=x^4-17 x^3+17 x^2-17 x+20 \text { tại } x=16 \text {. }$

Giải : Cách 1. Chú ý rằng $\mathrm{x}=16$ nên $\mathrm{x}-16=0$, do đó ta biến đổi để biểu thức $\mathrm{A}$ chứa nhiều biểu thức dạng $\mathrm{x}-16$.

$A =x^4-16 x^3-x^3+16 x^2+x^2-16 x-x+16+4 $

$=x^3(x-16)-x^2(x-16)+x(x-16)-(x-16)+4=4$

Cách 2. Trong biểu thức $\mathrm{A}$, ta thay các số 17 bởi $\mathrm{x}+1$, còn 20 thay bởi $\mathrm{x}+4$.

$A =x^4-x^3(x+1)+x^2(x+1)-x(x+1)+x+4$

$=x^4-x^4-x^3+x^3+x^2-x^2-x+x+4=4$

Ví dụ 2. Tìm ba số tự nhiên liên tiếp, biết rằng nếu cộng ba tích của hai trong ba số ấy, ta được 242 .

Giải : Gọi $\mathrm{x}-1, \mathrm{x}, \mathrm{x}+1$ là ba số tự nhiên liên tiếp. Ta có :

$x(x-1)+x(x+1)+(x-1)(x+1)=242$

Sau khi rút gọn ta được $3 x^2-1=242$ nên $x^2=81$.

Do $\mathrm{x}$ là số tự nhiên nên $\mathrm{x}=9 .$ Ba số tự nhiên phải tìm là $8 ; 9 ; 10$.

BÀI TẬP

– Nhân đơn thức với đa thức

1. Thực hiện phép tính :

a) $3 x^n\left(6 x^{n-3}+1\right)-2 x^n\left(9 x^{n-3}-1\right)$;

b) $5^{\mathrm{n}+1}-4.5^{\mathrm{n}}$

c) $6^2 \cdot 6^4-4^3\left(3^6-1\right)$

2. Tìm $\mathrm{x}$, biết :

a) $4(18-5 x)-12(3 x-7)=15(2 x-16)-6(x+14)$;

b) $5(3 x+5)-4(2 x-3)=5 x+3(2 x+12)+1$;

c) $2(5 x-8)-3(4 x-5)=4(3 x-4)+11$;

d) $5 x-3{4 x-2[4 x-3(5 x-2)]}=182$.

3. Tính giá trị của các biểu thức :

a) $A=x^3-30 x^2-31 x+1$ tại $x=31$

b) $\mathrm{B}=\mathrm{x}^5-15 \mathrm{x}^4+16 \mathrm{x}^3-29 \mathrm{x}^2+13 \mathrm{x}$ tại $\mathrm{x}=14$

c) $C=x^{14}-10 x^{13}+10 x^{12}-10 x^{11}+\ldots+10 x^2-10 x+10$ tại $x=9$.

4. Tính giá trị của biểu thức sau bằng cách thay số bởi chữ một cách hợp lí :

$A=2 \frac{1}{315} \cdot \frac{1}{651}-\frac{1}{105} \cdot 3 \frac{650}{651}-\frac{4}{315.651}+\frac{4}{105}$

– Nhân đa thức với đa thức

5. Thực hiện phép tính :

a) $(x-1)\left(x^5+x^4+x^3+x^2+x+1\right)$

b) $(x+1)\left(x^6-x^5+x^4-x^3+x^2-x+1\right)$.

6. Tìm x, biết :

a) $(x+2)(x+3)-(x-2)(x+5)=6$;

b) $(3 x+2)(2 x+9)-(x+2)(6 x+1)=(x+1)-(x-6)$;

c) $3(2 x-1)(3 x-1)-(2 x-3)(9 x-1)=0$.

7. Cho $a+b+c=0$. Chứng minh rằng $M=N=P$ với : $\mathrm{M}=\mathrm{a}(\mathrm{a}+\mathrm{b})(\mathrm{a}+\mathrm{c}) ; \quad \mathrm{N}=\mathrm{b}(\mathrm{b}+\mathrm{c})(\mathrm{b}+\mathrm{a}) ; \quad \mathrm{P}=\mathrm{c}(\mathrm{c}+\mathrm{a})(\mathrm{c}+\mathrm{b})$

8. Chứng minh các hằng đẳng thức :

a) $(x+a)(x+b)=x^2+(a+b) x+a b$

b) $(x+a)(x+b)(x+c)=x^3+(a+b+c) x^2+(a b+b c+c a) x+a b c .$

9. Cho $\mathrm{a}+\mathrm{b}+\mathrm{c}=2 \mathrm{p}$. Chứng minh hằng đẳng thức :

$2 b c+b^2+c^2-a^2=4 p(p-a)$

10. Xét các ví dụ : $53.57=3021, \quad 72.78=5616$.

Hãy xây dựng quy tắc nhân nhẩm hai số có hai chữ số, trong đó các chữ số hàng chục bằng nhau, còn các chữ số hàng đơn vị có tổng bằng $10 .$

11. Cho biểu thức

$M=(x-a)(x-b)+(x-b)(x-c)+(x-c)(x-a)+x^2$

Tính $\mathrm{M}$ theo $\mathrm{a}, \mathrm{b}, \mathrm{c}$, biết rằng $\mathrm{x}=\frac{1}{2} \mathrm{a}+\frac{1}{2} \mathrm{~b}+\frac{1}{2} \mathrm{c}$.

12. Cho dãy số $1,3,6,10,15, \ldots, \frac{n(n+1)}{2}, \ldots$

Chứng minh rằng tổng hai số hạng liên tiếp của dãy bao giờ cũng là số chính phương.

13. Số a gồm 31 chữ số 1 , số $\mathrm{b}$ gồm 38 chữ số 1 . Chứng minh rằng $\mathrm{ab}-2$ chia hết cho 3 .

$14^*$ . Số $3^{50}+1$ có là tích của hai số tự nhiên liên tiếp khônğ ?

15*. a) Thực hiện phép tính :

$A=\left(2^9+2^7+1\right)\left(2^{23}-2^{21}+2^{19}-2^{17}+2^{14}-2^{10}+2^9-2^7+1\right)$

b) Số $2^{32}+1$ có là số nguyên tố không ?

 

 

 

 

 

 

 

 

 

 

 

 

 

 

 

 

 

 

 

 

 

 

 

 

 

 

 

 

 

 

 

 

 

 

 

 

 

 

 

 

PHÉP NHÂN VÀ PHÉP CHIA ĐA THỨC – P.4

CHIA ĐA THỨC

 

Đa thức $\mathrm{A}(\mathrm{x})$ gọi là chia hết cho đa thức $\mathrm{B}(\mathrm{x})$ khác 0 nếu tồn tại đa thức $\mathrm{Q}(\mathrm{x})$ sao cho $\mathrm{A}(\mathrm{x})=\mathrm{B}(\mathrm{x}) \cdot \mathrm{Q}(\mathrm{x})$.

Người ta chứng minh được rằng : Với mọi cặp đa thức $\mathrm{A}(\mathrm{x})$ và $\mathrm{B}(\mathrm{x})$ trong đó $\mathrm{B}(\mathrm{x}) \neq 0$, tồn tại duy nhất cặp đa thức $\mathrm{Q}(\mathrm{x})$ và $\mathrm{R}(\mathrm{x})$ sao cho $\mathrm{A}(\mathrm{x})=\mathrm{B}(\mathrm{x}) \cdot \mathrm{Q}(\mathrm{x})+\mathrm{R}(\mathrm{x})$, trong đó $R(x)=0$ hoặc bậc của $R(x)$ nhỏ hơn bậc của $B(x)$.

Nếu $R(x)=0$ thì $A(x)$ chia hết cho $B(x)$. Nếu $R(x) \neq 0$ thì $A(x)$ không chia hết cho $B(x)$, khi đó $Q(x)$ là thương và $R(x)$ là dư của phép chia $A(x)$ cho $B(x)$.

Ví dụ 1. Tìm số tự nhiên $\mathrm{n}$ để đa thức $\mathrm{A}$ chia hết cho đơn thức $\mathrm{B}$ :

$A=3 x^{n-1} y^6-5 x^{n+1} y^4 ; B=2 x^3 y^n$

Tìm thương $\mathrm{A}: \mathrm{B}$ trong trường hợp đó.

Giải : Điều kiện để $\mathrm{A}$ chia hết cho $\mathrm{B}$ là :

$\left\{\begin{array}{r}\mathrm{n}-1 \geq 3 \\ \mathrm{n}+1 \geq 3 \\ 6 \geq \mathrm{n} \\ 4 \geq \mathrm{n}\end{array} \Leftrightarrow\left\{\begin{array}{l}\mathrm{n} \geq 4 \\ \mathrm{n} \leq 4\end{array} \Leftrightarrow \mathrm{n}=4\right.\right.$

Vậy với $\mathrm{n}=4$ thì đa thức $\mathrm{A}$ chia hết cho đơn thức $\mathrm{B}$. Khi đó

$A: B=\left(3 x^3 y^6-5 x^5 y^4\right):\left(2 x^3 y^4\right)=\frac{3}{2} y^2-\frac{5}{2} x^2$

Ví dụ 2. Xác định các số hữu tỉ a và $\mathrm{b}$ để đa thức $\mathrm{x}^3+\mathrm{ax}+\mathrm{b}$ chia hết cho đa thức $x^2+x-2$.

Giải : Cách 1. Đặt tính chia :

Để chia hết thì đa thức dư phải bằng 0 với mọi giá trị của $x$, nên :

$\left\{\begin{array}{l}a+3=0 \\ b-2=0\end{array} \Leftrightarrow\left\{\begin{array}{l}a=-3 \\ b=2\end{array}\right.\right.$

Vậy với $\mathrm{a}=-3 ; \mathrm{b}=2$ thì $\mathrm{x}^3+\mathrm{ax}+\mathrm{b}$ chia hết cho $\mathrm{x}^2+\mathrm{x}-2$.

Cách 2. (Phương pháp hệ số bất định)

Đa thức bị chia có bậc ba, đa thức chia có bậc hai nên thương là một nhị thức bậc nhất, hạng tử bậc nhất là $\mathrm{x}^3: \mathrm{x}^2=\mathrm{x}$.

Gọi thương là $\mathrm{x}+\mathrm{c}$, ta có :

$x^3+a x+b=\left(x^2+x-2\right)(x+c)$

nên

$x^3+a x+b=x^3+(c+1) x^2+(c-2) x-2 c $

Hai đa thức trên bằng nhau nên :

$\left\{\begin{array}{l}\mathrm{c}+1=0 \\ \mathrm{c}-2=\mathrm{a} \\ -2 \mathrm{c}=\mathrm{b}\end{array} \Leftrightarrow\left\{\begin{array}{l}\mathrm{c}=-1 \\ \mathrm{a}=-3 \\ \mathrm{~b}=2\end{array}\right.\right.$

Vậy với $\mathrm{a}=-3 ; \mathrm{b}=2$ thì $\mathrm{x}^3+\mathrm{ax}+\mathrm{b}$ chia hết cho $\mathrm{x}^2+\mathrm{x}-2$, thương là $\mathrm{x}-1$.

Cách 3. (Phương pháp xét giá trị riêng)

Gọi thương khi chia $\mathrm{x}^3+\mathrm{ax}+\mathrm{b}$ cho $\mathrm{x}^2+\mathrm{x}-2$ là $\mathrm{Q}(\mathrm{x})$, ta có :

$x^3+a x+b=(x-1)(x+2) Q(x)$

Vì đẳng thức đúng với mọi $x$ nên lần lượt cho $\mathrm{x}=1, \mathrm{x}=-2$, ta được :

$\left\{\begin{array}{l}1+a+b=0 \\ -8-2 a+b=0\end{array} \Leftrightarrow\left\{\begin{array}{l}a+b=-1 \\ -2 a+b=8\end{array} \Leftrightarrow\left\{\begin{array}{l}a=-3 \\ b=2 .\end{array}\right.\right.\right.$

Với $a=-3 ; b=2$ thì $x^3+a x+b$ chia hết cho $x^2+x-2$.

BÀI TẬP

Chia đơn thức cho đơn thức

71. Thực hiện phép tính :

a) $8^{12}: 4^6$;

b) $27^6: 9^2$;

c) $\frac{9^{15} \cdot 25^3 \cdot 4^3}{3^{10} \cdot 50^6}$

72. Chứng minh rằng biểu thức sau không âm với mọi giá trị của biến :

$A=\left(-15 x^3 y^6\right):\left(-5 x y^2\right)$

73. Chứng minh rằng giá trị của biểu thức sau không phụ thuộc vào giá trị của biến $\mathrm{y}(\mathrm{x} \neq 0 ; \mathrm{y} \neq 0)$ :

$B=\frac{2}{3} x^2 y^3:\left(-\frac{1}{3} x y\right)+2 x(y-1)(y+1)$

74. Tìm số tự nhiên $\mathrm{n}$ để đơn thức $\mathrm{A}$ chia hết cho đơn thức $\mathrm{B}$ :

$A=4 x^{n+1} y^2 ; B=3 x^3 y^{n-1}$

Chia đa thức cho dơn thức

75. Thực hiện phép tính :

a) $\left(\frac{1}{2} a^2 x^4+\frac{4}{3} a x^3-\frac{2}{3} a x^2\right):\left(-\frac{2}{3} a x^2\right)$

b) $4\left(\frac{3}{4} x-1\right)+\left(12 x^2-3 x\right):(-3 x)-(2 x+1)$.

76. Thực hiện phép tính rồi tìm giá trị nhỏ nhất của biểu thức :

$A=\left(9 x y^2-6 x^2 y\right):(-3 x y)+\left(6 x^2 y+2 x^4\right):\left(2 x^2\right) $

77. Tìm số tự nhiên $\mathrm{n}$ để đa thức $\mathrm{A}$ chia hết cho đơn thức $\mathrm{B}$ :

$A=7 x^{n-1} y^5-5 x^3 y^4 ; \quad B=5 x^2 y^n$

Chia đa thức cho đa thức

78. Rút gọn biểu thức

$\left[\left(x^3+y^3\right)-2\left(x^2-y^2\right)+3(x+y)^2\right]:(x+y)$

79. Chia các đa thức :

a) $\left(3 x^4-2 x^3-2 x^2+4 x-8\right):\left(x^2-2\right)$;

b) $\left(2 x^3-26 x-24\right):\left(x^2+4 x+3\right)$;

c) $\left(x^3-7 x+6\right):(x+3)$.

80. Xác định hằng số a sao cho :

a) $4 x^2-6 x+$ a chia hết cho $x-3$;

b) $2 \mathrm{x}^2+\mathrm{x}+\mathrm{a}$ chia hết cho $\mathrm{x}+3$;

c) $x^3+a x^2-4$ chia hết cho $x^2+4 x+4$.

81. Xác địṇh hằng số a sao cho :

a) $10 x^2-7 x+a$ chia hết cho $2 x-3$;

b) $2 x^2+a x+1$ chia cho $x-3$ dư 4 ;

c) $a x^5+5 x^4-9$ chia hết cho $x-1$.

82. Xác định các hằng số a và $\mathrm{b}$ sao cho :

a) $\mathrm{x}^4+\mathrm{ax}+\mathrm{b}$ chia hết cho $\mathrm{x}^2-4$;

b) $x^4+a x^3+b x-1$ chia hết cho $x^2-1$;

c) $x^3+a x+b$ chia hết cho $x^2+2 x-2$.

83. Xác định các hằng số a và b sao cho :

a) $x^4+a x^2+b$ chia hết cho $x^2-x+1$;

b) $a x^3+b x^2+5 x-50$ chia hết cho $x^2+3 x-10$;

c) $a x^4+b x^3+1$ chia hết cho $(x-1)^2$;

d) $x^4+4$ chia hết cho $x^2+a x+b$.

84. Tìm các hằng số $a$ và $b$ sao cho $x^3+a x+b$ chia cho $x+1$ thì dư 7 , chia cho $x-3$ thì dư $-5$.

85. Tìm các hằng số $\mathrm{a}, \mathrm{b}, \mathrm{c}$ sao cho $\mathrm{ax}^3+\mathrm{bx}^2+\mathrm{c}$ chia hết cho $\mathrm{x}+2$, chia cho $x^2-1$ thì dư $x+5$.

 

 

 

ĐỀ THI CHỌN ĐỘI TUYỂN HSG QUỐC GIA CỦA TRƯỜNG PTNK NĂM 2016 – 2017

ĐỀ THI

Ngày thi thứ nhất

Bài 1. Tìm tất cả $a$ để dãy số $\left(u_n\right)$ hội tụ, biết $u_1=a$ và $\forall n \in \mathbb{N}^*$ thì:

$\quad\quad\quad\quad\quad\quad\quad\quad\quad\quad u_{n+1}=\left\{\begin{array}{l}2 u_n-1 \text { nếu } u_n>0 \\ -1 \text { nếu }-1 \leq u_n \leq 0 \\ u_n^2+4 u_n+2 \text { nếu } u_n<-1\end{array}\right.$

Bài 2. Tìm số nguyên dương $k$ nhỏ nhất để bất đẳng thức

$\quad\quad\quad\quad\quad\quad\quad\quad\quad\quad x^k y^k z^k\left(x^3+y^3+z^3\right) \leq 3$

luôn đúng với mọi số thực dương $x, y, z$ thoả mãn điều kiện $x+y+z=3$.

Bài 3. Cho hàm số $f: \mathbb{N}^* \rightarrow \mathbb{N}^*$ thoả mãn hai điều kiện sau:

$\quad\quad$ i) $f$ là hàm tăng thật sự trên $\mathbb{N}^*$.

$\quad\quad$ ii) $f(2 n)=2 f(n) \forall n \in \mathbb{N}^*$.

(a) Giả sử $f(1)=3$ và $p>3$ là số nguyên tố. Chứng minh rằng tồn tại số nguyên dương $n$ sao cho $f(n)$ chia hết cho $p$.

(b) Cho $q$ là số nguyên tố lẻ. Hãy xây dựng một hàm $f$ thoả mãn các điều kiện của bài toán mà $f(n)$ không chia hết cho $q$ với mọi $n$ nguyên dương.

Bài 4. Cho tam giác $A B C$ có góc $\angle B A C$ tù và $A H \perp B C(H$ nằm trên $B C)$. Điểm $M$ thay đổi trên cạnh $A B$. Dựng điểm $N$ sao cho $\Delta B M N \sim \triangle H C A$, với $H$ và $N$ nằm khác phía đối với đường thẳng $A B$.

(a) Gọi $C M$ cắt đường tròn ngoại tiếp tam giác $B M N$ tại $K$. Chứng minh rằng $N K$ luôn đi qua một điểm cố định.

(b) Gọi $N H$ cắt $A C$ tại $P$. Dựng điểm $Q$ sao cho $\triangle H P Q \sim \triangle H N M$, với $Q$ và $M$ nằm khác phía đối với đường thẳng $N P$. Chứng minh rằng $Q$ luôn thuộc một đường thẳng cố định.

Ngày thi thứ hai

Bài 5. Với mỗi số nguyên dương $n$, tồn tại duy nhất số tự nhiên $a$ thoả mãn điều kiện $a^2 \leq n<(a+1)^2$. Đặt $\Delta_n=n-a^2$.

(a) Tìm giá trị nhỏ nhất của $\Delta_n$ khi $n$ thay đổi và luôn thoả mãn $n=15 m^2$ với $m$ là số nguyên dương.

(b) Cho $p, q$ là các số nguyên dương và $d=5(4 p+3) q^2$. Chứng minh rằng $\Delta_d \geq 5$.

Bài 6. Với các số nguyên $a, b, c, d$ thoả mãn $1 \leq a<b<c<d$, ký hiệu:

$T(a, b, c, d)=[(x, y, z, t) \subset \mathbb{N}^* \mid 1 \leq x<y<z<t, x \leq a, y \leq b, z \leq c, t \leq d]$

(a) Tình số phần tử của $T(1,4,6,7)$.

(b) Cho $a=1$ và $b \geq 4$. Gọi $d_1$ là số phần tử của $T(a, b, c, d)$ chứa 1 và không chứa $2 ; d_2$ là số phần tử chứa 1,2 và không chứa $3 ; d_3$ là số phần tử chứa $1,2,3$ và không chứa 4 . Chứng minh rằng $d_1 \geq 2 d_2-d_3$. Đẳng thức xảy ra khi nào?

Bài 7. Trong một hệ thống máy tính, một máy tính bất kỳ có kết nối trực tiếp với ít nhất $30 \%$ máy tính khác của hệ thống. Hệ thống này có một chương trình cảnh báo và ngăn chặn khá tốt, do đó khi một máy tính bị virus, nó chỉ có đủ thời gian lây cho các máy tính được kết nối trực tiếp với nó. Chứng minh rằng dù vậy, kẻ tấn công vẫn có thể chọn hai máy tính của hệ thống mà nếu thả virus vào hai máy đó, ít nhất $50 \%$ máy tính của hệ thống sẽ bị nhiễm virus.

Bài 8. Cho tam giác $A B C$ nhọn. Đường tròn $(I)$ có tâm $I$ thuộc cạnh $B C$ và tiếp xúc với các cạnh $A B, A C$ lần lượt tại $E, F$. Lấy $M, N$ bên trong tứ giác $B C E F$ sao cho $E F N M$ nội tiếp $(I)$ và các đường thẳng $M N, E F, B C$ dồng quy. Gọi $M F$ cắt $N E$ tại $P, A P$ cắt $B C$ tại $D$.

(a) Chứng minh rằng $A, D, E, F$ cùng thuộc một đường tròn.

(b) Lấy trên các đường thẳng $B N, C M$ các điểm $H, K$ sao cho $\angle A C H=$ $\angle A B K=90^{\circ}$. Gọi $T$ là trung điểm $H K$. Chứng minh rằng $T B=T C$.

 

LỜI GIẢI

Ngày thi thứ nhất

Bài 1. Tìm tất cả $a$ để dãy số $\left(u_n\right)$ hội tụ, biết $u_1=a$ và $\forall n \in \mathbb{N}^*$ thì:

$\quad\quad\quad\quad\quad\quad\quad\quad\quad\quad u_{n+1}=\left\{\begin{array}{l}2 u_n-1 \text { nếu } u_n>0, \\ -1 \text { nếu }-1 \leq u_n \leq 0, \\ u_n^2+4 u_n+2 \text { nếu } u_n<-1\end{array}\right.$

Lời giải. Có các trường hợp sau cần xem xét:

  • Nếu $a>1$, bằng quy nạp đơn giản, ta có $u_n>1 \forall n \in \mathbb{N}^*$ và

$\quad\quad\quad\quad\quad\quad\quad\quad\quad\quad u_n=2^{n-1}(a-1)+1, \forall n \in \mathbb{N}^* .$

Do $a>1$, cho $n \rightarrow+\infty$ thì $u_n \rightarrow+\infty$. Từ đó $\left(u_n\right)$ không hội tụ.

  • Nếu $a=1$ thì $u_n=1 \forall n \in \mathbb{N}^*$ hay $\left(u_n\right)$ hội tụ về 1 .

  • Nếu $0<a<1$, ta sẽ chứng minh rằng $\left(u_n\right)$ có ít nhất một số hạng không dương. Thật vậy, giả sử $u_n>0 \forall n \in \mathbb{N}^*$ thì theo trường hợp đầu tiên, ta có:

$\quad\quad\quad\quad\quad\quad\quad\quad\quad\quad u_n=2^{n-1}(a-1)+1 \forall n \in \mathbb{N}^*$

Do $a>1$, cho $n \rightarrow+\infty$ thì $u_n \rightarrow-\infty$, trái với việc $u_n>0 \forall n, \in \mathbb{N}^*$.

Từ đó điều giả sử là sai hay phải tồn tại $k \in \mathbb{N}^*\text { sao cho } u_k>0 \text { và } u_{k+1} \leq 0$. Với cách chọn chỉ số $\text{k}$ như vậy, ta có:

$\quad\quad\quad\quad\quad\quad\quad\quad\quad\quad -1 \leq 2 u_k-1=u_{k+1} \leq 0$

Khi đó $u_{k+2}=0$. Bằng quy nạp thì $u_n=-1 \forall n \in \mathbb{N}^*, n \geq k+2$. Điều này dễn đến $\left(u_n\right)$ hội tụ về $-1$.

  • Nếu $-1 \leq a \leq 0$, từ giả thiết thì $u_2=-1$. Bằng quy nạp thì $u_n=-1 \forall n \in$ $\mathbb{N}^*, n \geq 2$ hay $\left(u_n\right)$ hội tụ về $-1$.

  • Nếu $-2<a<-1$, ta có:

$\quad\quad\quad\quad\quad\quad\quad\quad\quad\quad u_2-u_1=a^2+3 a+2=(a+2)(a+1)<0$

Khi đó thì $u_2<u_1<-1$. Lại có $u_2=(a+2)^2-2 \geq-2$ nên $-2<u_2<-1$.

Bằng quy nạp, ta có $\left(u_n\right)$ là dãy giảm và $-2<u_n<-1$ nên $\left(u_n\right)$ hội tụ.

  • Nếu $-2-\sqrt{3} \leq a \leq-2$ thì $u_2=a^2-4 a+2$ và dễ có được:

$\quad\quad\quad\quad\quad\quad\quad\quad\quad\quad\quad\quad -1 \leq a^2-4 a+2 \leq 1$

Theo các trường hợp đã xét, dãy số $\left(u_n\right)$ hội tụ.

  • Nếu $a<-2-\sqrt{3}$, bằng vài tính toán, ta có $u^2=a^2-4 a+2>1$.

Theo trường hợp đầu tiên, dãy số $\left(u_n\right)$ không hội tụ.

Vậy dãy số $\left(u_n\right)$ hội tụ khi và chỉ khi $-2-\sqrt{3} \leq a \leq 1$.

Bài 2. Tìm số nguyên dương $k$ nhỏ nhất để bất đẳng thức

$\quad\quad\quad\quad\quad\quad\quad\quad\quad\quad x^k y^k z^k\left(x^3+y^3+z^3\right) \leq 3$

luôn đúng với mọi số thực dương $x, y, z$ thoả mãn điều kiện $x+y+z=3$.

Lời giải. Ta sẽ chứng minh rằng $k=3$ là số nguyên dương nhỏ nhất thoả mãn bài toán. Trước hết, chọn $x=y=\frac{3}{4}, z=\frac{3}{2}$ thì ta phải có:

$\quad\quad\quad\quad\quad\quad\quad\quad \left(\frac{3}{4}\right)^{2 k} \cdot\left(\frac{3}{2}\right)^k\left(2 \cdot\left(\frac{3}{4}\right)^3+\left(\frac{3}{2}\right)^3\right) \leq 3$

Dễ thấy đánh giá trên chỉ đúng nếu $k \geq 3$. Ta đưa về chứng minh rằng:

$\quad\quad\quad\quad\quad\quad\quad\quad\quad\quad x^3 y^3 z^3\left(x^3+y^3+z^3\right) \leq 3 .$

Không mất tính tổng quát, giả sử $x \geq y \geq z$ thì $z \leq 1$. Ta có:

$\quad\quad\quad\quad\quad x^3+y^3=(x+y)^3-3 x y(x+y)=(3-z)^3-3 x y(x+y) \text { hay } $

$\quad\quad\quad\quad\quad (3-z)^3+z^3 \leq \frac{3}{x^3 y^3 z^3}+3 x y(x+y)$

Khai triển và thu gọn, bất đẳng thức trở thành:

$\quad\quad\quad\quad\quad\quad\quad\quad\quad\quad 3 z^2-9 z+9 \leq \frac{1}{x^3 y^3 z^3}+x^2 y+x y^2$

Theo bất đẳng thức AM-GM, ta có vế phải của bất đẳng thức trên sẽ không nhỏ hơn $\frac{3}{z}$. Từ đây ta chỉ cần chứng minh rằng

$\quad\quad\quad\quad\quad\quad\quad 3 z^2-9 z+9 \leq \frac{3}{z} \text { hay } 3(z-1)^3 \leq 0 \text {, đúng. }$

Vậy $k=3$ là hằng số nguyên dương nhỏ nhất thoả mãn bài toán.

Nhận xét. Dưới đây là các cách xử lý khác cho bất đẳng thức ứng với $k=3$ ở trên.

Cách 1. Không mất tính tổng quát ta giả sử $x \leq y \leq z$. Khi đó luôn tồn tại $m>n \geq 0$ sao cho $x=m-n, y=m+n$. Khi đó

$\quad\quad\quad\quad\quad\quad\quad\quad\quad\quad z=3-2 m ; m=\frac{x+y}{2} \leq 1$

Xét hàm số

$f(n)=(m-n)^3(m+n)^3 z^3\left[z^3+(m-n)^3+(m+n)^3\right]=z^3\left(m^2-n^2\right)^3\left(z^3+2 m^3+6 m n^2\right)$

thì

$\quad\quad\quad\quad\quad f^{\prime}(n)=z^3\left(m^2-n^2\right)^2\left(-6 n z^3-48 m n^3\right) \leq 0$

nên

$\quad\quad\quad\quad\quad f(n) \leq f(0)=m^6 z^3\left(z^3+2 m^3\right)=m^6(3-2 m)^3\left((3-2 m)^3+2 m^3\right)$

Xét hàm số

$\quad\quad\quad\quad\quad\quad\quad\quad\quad\quad g(m)=m^6(3-2 m)^3\left[(3-2 m)^3+2 m^3\right]$

thì

$\quad\quad\quad\quad\quad g^{\prime}(m)=18 m^5(3-2 m)^2(m-1)\left[(m-1)\left(8 m^2-37 m+26\right)-1\right] \geq 0 .$

Vậy nên $g(m) \leq g(1)=3$, bài toán được giải quyết.

Cách 2. Không mất tính tổng quát, ta giả sử $z$ là số lớn nhất trong ba số $x, y, z$. Đặt $t=\frac{x+y}{2}$ và $f(x, y, z)=x^3 y^3 z^3\left(x^3+y^3+z^3\right)$. Ta sẽ chứng minh $f(x, y, z) \leq$ $f(t, t, z)$. Ta có

$\quad\quad\quad\quad f(t, t, z)-f(x, y, z)=z^3\left[t^6\left(2 t^3+z^3\right)-x^3 y^3\left(x^3+y^3+z^3\right)\right] .$

Mặt khác,

$t^6\left(2 t^3+z^3\right)-x^3 y^3\left(x^3+y^3+z^3\right)=z^3\left(t^6-x^3 y^3\right)+2 t^9-x^3 y^3(x+y)\left(x^2+y^2-x y\right) $

$=z^3\left(t^6-x^3 y^3\right)+2 t^9-2 t x^3 y^3\left(4 t^2-3 x y\right) \geq t^3\left(t^6-x^3 y^3\right)+2 t^9-2 t x^3 y^3\left(4 t^2-3 x y\right) $

$=3 t\left(t^2-x y\right)\left[t^6+x y\left(2 x y+t^2\right)\left(t^2-x y\right)\right] \geq 0 .$

Vậy nên

$\quad\quad\quad\quad\quad f(x, y, z) \leq f(t, t, z)=f(t, t, 3-2 t)=t^6(3-2 t)^3\left[2 t^3+(3-2 t)^3\right]$

Ta chỉ cần chứng minh

$\quad\quad\quad\quad\quad\quad\quad\quad\quad\quad t^6(3-2 t)^3\left[2 t^3+(3-2 t)^3\right] \leq 3$

Đến đây thực hiện như cách 1 ở trên.

Bài 3. Cho hàm số $f: \mathbb{N}^* \rightarrow \mathbb{N}^*$ thoả mãn hai điều kiện sau:

i) $f$ là hàm tăng thật sự trên $\mathbb{N}^*$.

ii) $f(2 n)=2 f(n) \forall n \in \mathbb{N}^*$.

(a) Giả sử $f(1)=3$ và $p>3$ là số nguyên tố. Chứng minh rằng tồn tại số nguyên dương $n$ sao cho $f(n)$ chia hết cho $p$.

(b) Cho $q$ là số nguyên tố lẻ. Hãy xây dựng một hàm $f$ thoả mãn các điều kiện của bài toán mà $f(n)$ không chia hết cho $q$ với mọi $n$ nguyên dương.

Lời giải. (a) Đặt $A=[f(n+1)-f(n) \mid n \in \mathbb{N}^*].$

Vì $\text { f là hàm số tăng thực sự trên } \mathbb{N}^* \text { nên } A \subset \mathbb{N}^*$.

Khi đó phải tồn tại $k=\min A \text { và tồn tại } n \in \mathbb{N}^* \text { để } k=f(n+1)-f(n)$. Khi đó:

$\quad\quad\quad\quad\quad\quad\quad\quad\quad\quad f(2 n+2)-f(2 n)=2 f(n+1)-2 f(n)=2 k .$

Lại có $f(2 n+2)-f(2 n+1), f(2 n+1)-f(2 n) \geq k$ nên

$\quad\quad\quad\quad\quad\quad\quad\quad\quad\quad f(2 n+2)-f(2 n+1)+f(2 n+1)-f(2 n) \geq 2 k .$

Từ đây ta phải có $f(2 n+2)-f(2 n+1)=f(2 n+1)-f(2 n)=k$. Bằng quy nạp theo $m$, ta chứng minh được

$\quad\quad\quad\quad\quad\quad\quad\quad\quad\quad f\left(2^m n+t\right)=2^m f(n)+t k \forall t, m \in \mathbb{N}, t \leq m .$

Lại có $f(1)=3, f(2)=6$ nên $k \leq 3<p$ hay $(k, p)=1$.

Xét $p$ số nguyên dương sau:

$\quad\quad\quad\quad\quad\quad\quad\quad\quad\quad f\left(2^p n\right), f\left(2^p n+1\right), f\left(2^p n+2\right), \ldots, f\left(2^p n+p-1\right)$

lập thành một cấp số cộng có công sai $k$ nên là một hệ thặng dư đầy đủ modulo $p$. Từ đó phải tồn tại một số hạng chia hết cho $p$.

(b) Ta xây dựng một hàm số $f$ với các điều kiện như sau:

$\quad\quad$ i) $f(1)=2^a>q\left(a \in \mathbb{N}^*\right.$,

$\quad\quad$ ii) $f(2 n)=2 f(n) \forall n \in \mathbb{N}^*$,

$\quad\quad$ iii) $f(2 n+1)=f(2 n)+q \forall n \in \mathbb{N}^*$.

Ta chứng minh rằng hàm số $f$ vừa xây dựng thỏa mãn bài toán.

Trước hết ta chứng minh rằng $f$ là hàm tăng thực sự, cụ thể là:

$\quad\quad\quad\quad\quad\quad\quad\quad\quad\quad f(n+1)-f(n) \geq q \forall n \in \mathbb{N}^* .$

Với $n=1$, ta có $f(2)-f(1)=2.2^a-2^a=2^a>q$. Giả sử khẳng định cần chứng minh đúng đến $n=k$. Xét các khả năng sau:

  • Nếu $k$ là số chẵn, ta có $f(k+1)=f(k)+q$ thỏa mãn yêu cầu.

  • Nếu $k$ là số lẻ, ta có:

$\quad\quad\quad\quad f(k+1)=2 f\left(\frac{k+1}{2}\right) \geq 2\left(f\left(\frac{k-1}{2}\right)+q\right)=f(k-1)+2 q .$

Lại có $f(k)=f(k-1)+q$ nên $f(k+1) \geq f(k)+q$.

Theo nguyên lý quy nạp, ta có $f(n+1)-f(n) \geq q \forall n \in \mathbb{N}^*$.

Bây giờ ta chứng minh rằng không tồn tại $n$ để $q \mid f(n)$. Trước hết thì $f(1)=2^a$ không chia hết cho $q$. Giả sử điều này đúng đến $n=k$. Xét các khả năng sau:

  • Nếu $k$ chẵn thì $f(k+1)=f(k)+q$ không chia hết cho $q$.

  • Nếu $k$ lẻ thì $f(k+1)=2 f\left(\frac{k+1}{2}\right)$ không chia hết cho $q$.

Theo nguyên lý quy nạp, $f(n)$ không chia hết cho $q$ với mọi $n \in \mathbb{N}^*$. Các điều kiện đã được kiểm tra đầy đủ.

Bài 4. Cho tam giác $A B C$ có góc $\angle B A C$ tù và $A H \perp B C(H$ nằm trên $B C$ ). Điểm $M$ thay đổi trên cạnh $A B$. Dựng điểm $N$ sao cho $\Delta B M N \sim \triangle H C A$, với $H$ và $N$ nằm khác phía đối với đường thẳng $A B$.

(a) Gọi $C M$ cắt đường tròn ngoại tiếp tam giác $B M N$ tại $K$. Chứng minh rằng $N K$ luôn đi qua một điểm cố định.

(b) Gọi $N H$ cắt $A C$ tại $P$. Dựng điểm $Q$ sao cho $\triangle H P Q \sim \Delta H N M$, với $Q$ và $M$ nằm khác phía đối với đường thẳng $N P$. Chứng minh rằng $Q$ luôn thuộc một đường thẳng cố định.

Lời giải. (a) Xét điểm $X$ trên $A C$ sao cho $\angle X B C=90^{\circ}$ và $K^{\prime}$ là giao điểm của $N X$ và $C M$. Ta có $\Delta B M N \sim \triangle B C X$ (cùng hướng). Từ đó có một phép vị tự quay tâm $B$ biến $M \mapsto N, C \mapsto X$.

Giả sử $C M$ cắt $B X$ tại $K^{\prime}$ thì $K^{\prime}$ thuộc đường tròn ngoại tiếp tam giác $B M N$. Từ đó $K^{\prime} \equiv K$ nên $N K$ luôn đi qua điểm $X$ cố định.

(b) Xét phép vị tự tâm $H$ biến

$\quad\quad\quad\quad\quad\quad\quad\quad\quad\quad N \mapsto P, M \mapsto Q, B \mapsto F .$

Ta có $\Delta B M N \sim \triangle F Q P$. Khi đó

$\quad\quad\quad\quad\quad\quad\quad\quad\quad\quad \angle F Q P=\angle B M N=\angle A C B=\angle F C P$

nên tứ giác $C F P Q$ nội tiếp. Từ đây dẫn đến

$\quad\quad\quad\quad\quad\quad\quad\quad\quad\quad \angle Q C P=\angle Q F P=\angle M B N=90^{\circ} .$

Vậy $Q$ thuộc đường thẳng qua $C$ vuông góc với $A C$, là đường thẳng cố định.

 

Ngày thi thứ hai

Bài 5. Với mỗi số nguyên dương $n$, tồn tại duy nhất số tự nhiên $a$ thoả mãn điều kiện $a^2 \leq n<(a+1)^2$. Đặt $\Delta_n=n-a^2$.

(a) Tìm giá trị nhỏ nhất của $\Delta_n$ khi $n$ thay đổi và luôn thoả mãn $n=15 m^2$ với $m$ là số nguyên dương.

(b) Cho $p, q$ là các số nguyên dương và $d=5(4 p+3) q^2$. Chứng minh rằng $\Delta_d \geq 5$.

Lời giải. (a) Ta cần tìm $\Delta_n$ nhỏ nhất để phương trình $15 m^2-a^2=\Delta_n$ có nghiệm nguyên dương. Nhận thấy $15-3^2=6$ nên $\min \Delta_n \leq 6$. Ta chứng minh rằng phương trình trên không có nghiệm nguyên dương với $\Delta_n<6$.

Ta có $3 \mid a^2+\Delta_n$. Suy ra $3 \mid \Delta_n$ hoặc $3 \mid \Delta_n+1$. Mặt khác $5 \mid a^2+\Delta_n$ nên $\Delta_n$ chia 5 chỉ có thể dư 0,1 hoặc 4 .

Từ đó nếu tồn tại $n$ để $\Delta_n<6$ thỏa mãn bài toán thì $\Delta_n=5$. Giả sử rằng tồn tại $n$ như thế, ta có $15 m^2-a^2=5$ hay $5 \mid a$. Đặt $a=5 s\left(s \in \mathbb{N}^*\right)$, ta có:

$\quad\quad\quad\quad\quad\quad\quad\quad\quad\quad\quad\quad 3 m^2-5 s^2=1 \text {. }$

Từ đó thì

$\quad\quad\quad\quad 3\left(m^2+s^2\right) \equiv 1 \quad(\bmod 8)$ hay $m^2+s^2 \equiv 3 \quad(\bmod 8)$

Điều này vô lý do $m^2$ chia 8 dư $0,1,4$. Vậy $\Delta_n$ nhỏ nhất là 6 .

(b) Ta có

$\quad\quad\quad\quad\quad\quad\quad\quad\quad\quad 5(4 p+3) q^2-a^2=\Delta_d .$

Do $a^2$ chia 5 dư $0,1,4$ nên $\Delta_d$ chia 5 dư $0,1,4$. Giả sử rằng có bộ số để $\Delta_d<5$. Xét các khả năng sau:

  • Nếu $\Delta_d=0$ thì $5(4 p+3) q^2=a^2$. Xét bộ số $(q, a)$ với $q+a$ nhỏ nhất. Từ phương trình trên, ta có $a^2+q^2 \equiv 0(\bmod 4)$ hay $a \equiv q \equiv 0(\bmod 2)$.

Đặt $a=2 a_1$ và $q=2 q_1$ với $a_1, q_1 \in \mathbb{N}^*$ thì bộ số $\left(q_1, a_1\right)$ cũng thoả mãn điều kiện $5(4 p+3) q_1^2=a_1^2$. Hơn nữa $q_1+a_1<q+a$, mâu thuẫn.

  • Nếu $\Delta_d=1$, ta có $a^2+1=5(4 p+3) q^2$. Do $5(4 p+3) \equiv 3(\bmod 4)$ nên số này tồn tại một ước nguyên tố $r \equiv 3(\bmod 4)$.

Do đó $a^2+1 \equiv 0(\bmod r)$ hay $r \mid 1$, vô lý.

  • Nếu $\Delta_d=4$, chứng minh tương tự, ta cũng có điều mâu thuẫn.

Vậy ta phải có $\Delta_d \geq 5$.

Bài 6. Với các số nguyên $a, b, c, d$ thoả mãn $1 \leq a<b<c<d$, ký hiệu: $T(a, b, c, d)=[(x, y, z, t) \subset \mathbb{N}^* \mid 1 \leq x<y<z<t, x \leq a, y \leq b, z \leq c, t \leq d]$.

(a) Tính số phần tử của $T(1,4,6,7)$.

(b) Cho $a=1$ và $b \geq 4$. Gọi $d_1$ là số phần tử của $T(a, b, c, d)$ chứa 1 và không chứa $2 ; d_2$ là số phần tử chứa 1,2 và không chứa $3 ; d_3$ là số phần tử chứa $1,2,3$ và không chứa 4 . Chứng minh rằng $d_1 \geq 2 d_2-d_3$. Đẳng thức xảy ra khi nào ?

Lời giải. (a) Với $T(1,4,6,7)$, ta có $x \leq 1$ nên $x=1$. Khi đó ta có $2 \leq y \leq 4$ hay $y \in{2,3,4}$. Xét các khả năng sau:

  • Nếu $y=2$ thì $3 \leq z \leq 6$. Với mỗi giá trị của $z$, ta có thể thu được $7-z$ giá trị của $t$ nên ta có 10 bộ số.

  • Nếu $y=3$, tương tự ta có 6 bộ số.

  • Nếu $y=4$, tương tự ta có 3 bộ số.

Vậy có tất cả 19 bộ số trong $T(1,4,6,7)$.

(b) Đặt các tập hợp sau:

$\quad\quad\quad\quad\quad\quad\quad \left\{\begin{array}{l}T_1={(1, y, z, t) \mid 3 \leq y \leq b, y<z \leq c, z<t \leq d} \\ T_2={(1,2, z, t) \mid 4 \leq z \leq c, z<t \leq d} \\ T_3={(1,2,3, t) \mid 5 \leq t \leq d}\end{array}\right.$

Ta có $d_3=\left|T_3\right|=d-4$ và

$\quad\quad\quad\quad\quad\quad\quad\quad\quad\quad d_2=\sum_{z=4}^c(d-z)=(c-3) d+\frac{(c+4)(c-3)}{2}$

Tiếp theo ta tính $d_1=\left|T_1\right|$. Vì $b \geq 4$ nên $y \geq 3$. Xét các khả năng sau

  • Nếu $y=3$ thì $T(1,3, z, t)=d_2$.

  • Nếu $y=4$ thì $T(1,4, z, t)=\sum_{z=5}^c(d-z)=(c-4) d-\frac{(c+5)(c-4)}{2}$.

Từ đó $d_1 \geq d_2+(c-4) d-\frac{(c+5)(c-4)}{2}$. Do đó, kết hợp với việc tính được giá trị của $d_2$, khi cộng theo vế thì $d_1+d_3-2 d_2 \geq 0$.

Vậy $d_1 \geq 2 d_2-d_3$. Đẳng thức xảy ra khi và chỉ khi $b=4$.

Nhận xét. Ngoài lời giải khá “đại số” phía trên, có một lời giải khác cho ý sau của bài toán sử dụng song ánh:

  • Điểm mấu chốt là phân rã $T_1, T_2, T_3$ thành các nhóm thích hợp và thiết lập được đơn ánh giữa chúng. Với các tập $T_1, T_2, T_3$ định nghĩa như trên, ta viết $T_1$ thành $A \cup B \cup C$ có giao đôi một khác rỗng, trong đó

$\quad\quad\quad\quad\quad\quad \left\{\begin{array}{l}A={(1,3,4, t) \mid 5 \leq t \leq d} \\ B={(1,3, z, t) \mid 5 \leq z \leq c, z<t \leq d} \\ C={(1, y, z, t) \mid 4 \leq y \leq b, y<z \leq c, z<t \leq d}\end{array}\right.$

  • Dễ kiểm chứng rằng có song ánh từ $A$ vào $T_3$ nên $|A|=\left|T_3\right|=d_3$.

  • Xét $D={(1,4, z, t) \mid 5 \leq z \leq c, z<t \leq d}$. Dễ kiểm chứng rằng $D \subset C$ và có song ánh từ $D$ vào $B$ nên $|D|=|B|$.

  • Ta có $A \cup B={(1,3, z, t) \mid 4 \leq z \leq c, z<t \leq d}$. Dễ kiểm chứng rằng có song ánh từ $A \cup B$ vào $T_2$ nên $|A \cup B|=\left|T_2\right|=d_2$. Chú ý rằng $A \cap B=\varnothing$ nên $|A|+|B|=d_2$ hay $|B|=d_2-d_3$. Từ đó ta có:

$\quad\quad\quad\quad\quad d_1=|A|+|B|+|C| \geq|A|+|B|+|D|=d_3+2|B|$

Vậy $d_1 \geq d_3+2\left(d_2-d_3\right)=2 d_2-d_3$. Đẳng thức xảy ra khi và chỉ khi $b=4$.

Bài 7. Trong một hệ thống máy tính, một máy tính bất kỳ có kết nối trực tiếp với ít nhất $30 \%$ máy tính khác của hệ thống. Hệ thống này có một chương trình cảnh báo và ngăn chặn khá tốt, do đó khi một máy tính bị virus, nó chỉ có đủ thời gian lây cho các máy tính được kết nối trực tiếp với nó. Chứng minh rằng dù vậy, kẻ tấn công vẫn có thể chọn hai máy tính của hệ thống mà nếu thả virus vào hai máy đó, ít nhất $50 \%$ máy tính của hệ thống sẽ bị nhiễm virus.

Lời giải Trước hết ta chứng minh bổ đề sau: Xét một tập con $S$ bất kỳ của tập các máy tính $X$, khi đó tồn tại 1 máy tính của hệ thống kết nối trực tiếp với ít nhất $30 \%$ máy tính của $S$.

Thật vậy, xét các cặp $(s, x)$ với $s \in S, x \in X$ và $(s, x)$ kết nối trực tiếp với nhau. Khi đó, nếu tính theo $s$ thì số cặp như vậy sẽ không ít hơn $\frac{3}{10}|S||X|$. Do đó nếu tính theo $x$ thì sẽ phải tồn tại máy tính $x$ kết nối trực tiếp với ít nhất $\frac{3}{10}|S|$.

Quay trở lại bài toán,

Giả sử hệ thống có $n$ máy tính. Xét máy tính $A$ bất kỳ. Gọi $S$ là tập hợp các máy tính không kết nối trực tiếp với $A$. Nếu $S=\varnothing$ thì kết quả bài toán là hiển nhiên. Nếu $S \neq \varnothing$ thì theo bổ đề, tồn tại máy tính $B$ kết nối trực tiếp với ít nhất $30 \%$ máy tính trong $S$. Ta chứng minh hai máy tính $A$ và $B$ thỏa mãn yêu cầu bài toán.

Thật vậy, giả sử $A$ kết nối trực tiếp với $k$ máy tính khác. Khi đó, theo cách chọn, $A$ và $B$ sẽ kết nối trực tiếp với ít nhất

$\quad\quad\quad\quad\quad k+0,3(n-k)=0,7 k+0,3 n \geq 0,7 \cdot 0,3 n+0,3 n=0,51 n .$

Từ đây ta có được kết luận của bài toán.

Bài 8 . Cho tam giác $A B C$ nhọn. Đường tròn $(I)$ có tâm $I$ thuộc cạnh $B C$ và tiếp xúc với các cạnh $A B, A C$ lần lượt tại $E, F$. Lấy $M, N$ bên trong tứ giác $B C E F$ sao cho $E F N M$ nội tiếp $(I)$ và các đường thẳng $M N, E F, B C$ đồng quy. Gọi $M F$ cắt $N E$ tại $P, A P$ cắt $B C$ tại $D$.

(a) Chứng minh rằng $A, D, E, F$ cùng thuộc một đường tròn.

(b) Lấy trên các đường thẳng $B N, C M$ các điểm $H, K$ sao cho $\angle A C H=$ $\angle A B K=90^{\circ}$. Gọi $T$ là trung điểm $H K$. Chứng minh rằng $T B=T C$.

Lời giải. (a) Ta sẽ chứng minh rằng $A D \perp B C$. Gọi $X$ là điểm đồng quy của $E F, M N, B C$. Do $A E, A F$ tiếp xúc với $(I)$ nên $E F$ là đường đối cực của $A$ đối với (I). Ta có $X \in E F$ nên theo định lý La Hire, điểm $A$ sẽ nằm trên đường đối cực của $X$ đối với đường tròn $(I)$.

Lại có $P$ là giao điểm của $E N, F M$ nên $P$ nằm trên đường đối cực của $X$ đối với $(I)$. Vì thế nên $A P$ là đường đối cực của $X$ đối với $(I)$ hay $A P \perp B C$. Do đó

$\quad\quad\quad\quad\quad\quad\quad\quad\quad\quad \angle A D I=\angle A E I=\angle A F I=90^{\circ} .$

Vậy $A, D, E, F$ cùng thuộc một đường tròn.

(b) Gọi $S$ là giao điểm của $B N, C M$. Xét hai tam giác $P E F, S B C$ có $P E$ cắt $S B$ tại $N, P F$ cắt $S C$ tại $M, E F$ cắt $B C$ tại $X$ và $X, M, N$ thẳng hàng. Theo định lý Desargues thì $P S, E B, F C$ đồng quy. Mặt khác $E B$ cắt $F C$ tại $A$ nên $A, P, S$ thẳng hàng, dẫn đến $S \in A D$.

Tiếp theo ta sẽ chứng minh rằng $\angle B A K=\angle C A H$. Áp dụng định lý Ceva dạng lượng giác cho tam giác $A B C$ với:

  • Các đường thẳng $A D, B H, C K$ đồng quy:

$\quad\quad\quad\quad\quad\quad\quad\quad\quad\quad \frac{\sin \angle D A B}{\sin \angle D A C} \cdot \frac{\sin \angle H B C}{\sin \angle H B A} \cdot \frac{\sin \angle K C A}{\sin \angle K C B}=1$

  • Các đường thẳng $A H, B H, C H$ đồng quy:

$\quad\quad\quad\quad\quad\quad\quad\quad\quad\quad \frac{\sin \angle H A B}{\sin \angle H A C} \cdot \frac{\sin \angle H B C}{\sin \angle H B A} \cdot \frac{\sin \angle H C A}{\sin \angle H C B}=1$

  • Các đường thẳng $A K, B K, C K$ đồng quy:

$\quad\quad\quad\quad\quad\quad\quad\quad\quad\quad \frac{\sin \angle K A B}{\sin \angle K A C} \cdot \frac{\sin \angle K B C}{\sin \angle K B A} \cdot \frac{\sin \angle K C A}{\sin \angle K C B}=1$

Chú ý rằng do các góc vuông và góc bù nhau nên ta có

$\quad\quad\quad\quad\quad\quad\quad\quad\quad\quad \frac{\sin \angle H A C}{\sin \angle H A B}=\frac{\sin \angle K A B}{\sin \angle K A C}$

Từ đó sử dụng công thức cộng cho mẫu thức và biến đổi thì:

$\quad\quad\quad\quad\quad\quad\quad\quad\quad\quad \tan \angle H A C=\tan \angle K A B$

Dẫn đến $\angle H A C=\angle K A B$. Cuối cùng, ta sẽ chứng minh $T B=T C$.

Gọi $U, V$ lần lượt là trung điểm của các đoạn $A K, A H$. Ta có:

$\quad\quad\quad\quad\quad\quad\quad\quad\quad\quad U B=\frac{A K}{2}=V T, U T=\frac{A H}{2}=V C .$

Đồng thời, ta cũng có:

$\quad\quad\quad\quad\quad\quad \angle B U T=\angle B U A-\angle A U T=\angle A V C-\angle A V T=\angle T V C$

Do đó $\Delta B U T=\Delta T V C$ (c.g.c), vậy nên $T B=T C$.

Nhận xét. Để chứng minh $\angle H A C=\angle K A B$, cũng là mấu chốt của lời giải trên, ta có thể dùng bổ đề sau:

Cho tam giác $A B C$ có hai điểm $P, Q$ sao cho $A P, A Q$ đẳng giác trong góc $A$. Gọi $X$ là giao điểm của $B P, C Q$ và $Y$ là giao điểm của $B Q, C P$. Khi đó, ta cũng có $A X, A Y$ đẳng giác trong góc $A$.

 

 

 

 

 

 

 

 

 

 

 

 

 

 

 

 

 

 

 

 

 

 

 

 

 

 

 

 

 

 

 

 

 

 

 

 

 

 

 

 

 

 

 

 

 

 

 

 

 

 

 

 

 

 

 

 

 

 

 

 

 

Đề thi và đáp án kì thi chọn đội tuyển thi Quốc gia trường Phổ thông Năng khiếu năm học 2010 – 2011

ĐỀ THI

Ngày thi thứ nhất

Bài 1. Giải hệ phương trình sau:

$\quad\quad\quad\quad\quad\quad\quad\quad\quad\quad \left\{\begin{array}{l}\frac{5(x+y)}{x+y+6 x y}+\frac{6(x+z)}{x+z+5 x z}=4 \\ \frac{6(y+z)}{z+y+4 z y}+\frac{4(x+y)}{x+y+6 x y}=5 \\ \frac{4(x+z)}{x+y+5 x z}+\frac{5(y+z)}{y+z+4 y z}=6\end{array}\right.$

Bài 2. Tìm tất cả các hàm số $f: \mathbb{R} \rightarrow \mathbb{R}$ thỏa mãn:

$\quad\quad\quad\quad\quad\quad\quad\quad\quad\quad  f(|x|+y+f(y+f(y)))=3 y+|f(x)|, \forall x, y \in \mathbb{R}$

Bài 3. Cho $p$ là số nguyên tố lẻ và $n=2 p+r$ với $r \in{0,1,2, \ldots, p-1}$. Đặt $X={1,2, \ldots, n}$. Ánh xạ $f: X \rightarrow X$ được gọi là có tính chất $\mathcal{P}$ nếu $f$ không phải là ánh xạ đồng nhất và $f(f(\ldots(f(k)) \ldots)$ ) $=k$ (ánh xạ hợp $p$ lần) với mọi $k \in X$.

Đặt $A_f={k \in X \mid f(k)=k}$.

a) Chứng minh rằng nếu $f$ có tính chất $\mathcal{P}$ thì $\left|A_f\right| \equiv r(\bmod p)$.

b) Gọi $d$ là số các ánh xạ có tính chất $\mathcal{P}$. Chứng minh rằng $d$ không là ước của $n$ !.

(Kí hiệu $|A|$ chỉ số lượng các phần tử của tập hợp $A$.)

Bài 4. Cho tam giác $A B C$ nội tiếp đường tròn $(O)$ có $A$ cố định và $B, C$ thay đổi trên $(O)$ sao cho $B C$ luôn song song với một đường thẳng cố định. Các tiếp tuyến của $(O)$ tại $B$ và $C$ cắt nhau tại $K$. Gọi $M$ là trung điểm của $B C, N$ là giao điểm của $A M$ với $(O)$. Chứng minh rằng đường thẳng $K N$ luôn đi qua một điểm cố định.

Ngày thi thứ hai

Bài 5. Chứng minh rằng nếu $a, b, c$ là độ dài ba cạnh của một tam giác thì:

$\quad\quad\quad\quad\quad\quad\quad\quad\quad\quad (2 a+2 b-c)(2 b+2 c-a)(2 c+2 a-b)>25 a b c$

Bài 6. Cho dãy số $\left(u_n\right)$ thoả mãn $u_1=\sqrt{2}$ và $u_{n+1}=\frac{2 u_n^2+5 u_n+5}{2 u_n+4}, \forall n \geq 1$. Tìm $\lim \frac{u_n^2-3 u_n+5}{3 n^2+4 n-1}$.

Bài 7. Xét số tự nhiên $n>1$. Bắt đầu từ bộ số $1,2, \ldots, 2 n-1,2 n$, ta thực hiện phép biến đổi sau: Chọn hai số $a, b$ sao cho $a-b>1$, xoá hai số này và thay thế bởi hai số $a-1, b+1$. Với bộ số mới, ta lại tiếp tục thực hiện phép biến đổi tương tự’

a) Chứng minh rằng ta sẽ đạt đến trạng thái dừng, tức là không thể tiếp tục thực hiện phép biến đổi như vậy được nữa.

b) Gọi $k$ là số lần phép biến đổi cần thực hiện để đạt đến trạng thái dừng. Tìm giá trị nhỏ nhất và lớn nhất của $k$.

Bài 8. Cho đường tròn $\left(\gamma_1\right)$ đường kính $A B$ và đường tròn $\left(\gamma_2\right)$ tâm $A$ cắt $\left(\gamma_1\right)$ tại $C, D$. Điểm $M$ thay đổi trên cung $C D$ (nằm bên trong $\left(\gamma_1\right)$ ) của $\left(\gamma_2\right)$. Gọi $B M$ cắt $\left(\gamma_1\right)$ tại $N$ khác $M$ và $B$. Tìm giá trị nhỏ nhất của $\frac{N D+N C}{M N}$.

 

LỜI GIẢI

Ngày thi thứ nhất

Bài 1. Giải hệ phương trình

$\quad\quad\quad\quad\quad\quad\quad\quad\quad\quad \left\{\begin{array}{l}\frac{5(x+y)}{x+y+6 x y}+\frac{6(x+z)}{x+z+5 x z}=4 \\ \frac{6(y+z)}{z+y+4 z y}+\frac{4(x+y)}{x+y+6 x y}=5 \\ \frac{4(x+z)}{x+y+5 x z}+\frac{5(y+z)}{y+z+4 y z}=6\end{array} .\right.$

Lời giải. Đặt $u=\frac{x+y}{x+y+6 x y}, v=\frac{y+z}{y+z+4 y z}, w=\frac{z+x}{z+x+5 z x}$ thì ta có hệ

$\quad\quad\quad\quad\quad\quad\quad\quad\quad\quad \left\{\begin{array} { l }{ 5 u + 6 w = 4 } \\ { 6 v + 4 u = 5 } \\ { 4 w + 5 v = 6 }\end{array} \Leftrightarrow \left\{\begin{array}{l}8 u=1 \\ 4 v=3 \\ 16 w=9\end{array} .\right.\right.$

Suy ra

$\quad\quad\quad\quad\quad\quad\quad\quad\quad\quad \left\{\begin{array} { l }{ 7 ( x + y ) = 6 x y } \\ { 3 ( y + z ) = 1 2 y z } \\ { 7 ( z + x ) = 4 5 z x }\end{array} \Leftrightarrow \left\{\begin{array}{l}a+b=\frac{6}{7} \\ b+c=12 \\ c+a=\frac{45}{7}\end{array}\right.\right.$

trong đó $a=\frac{1}{x}, b=\frac{1}{y}, c=\frac{1}{z}$. Giải hệ trên, ta thu được $a=-\frac{33}{14}, b=\frac{45}{14}, c=\frac{123}{14}$ nên $(x, y, z)=\left(-\frac{14}{33}, \frac{14}{45}, \frac{14}{123}\right)$.

Bài 2. Tìm tất cả các hàm số $f: \mathbb{R} \rightarrow \mathbb{R}$ thỏa mãn:

$\quad\quad\quad\quad\quad\quad f(|x|+y+f(y+f(y)))=3 y+|f(x)|, \forall x, y \in \mathbb{R}$

Lời giải. Dễ thấy $f$ toàn ánh. Giả sử $f(a)=0$ và thay $x=0, y=a$, ta có

$\quad\quad\quad\quad\quad\quad\quad\quad\quad\quad 0=3 a+|f(0)|$

Suy ra $a$ tồn tại duy nhất và $a=-\frac{1}{3}|f(0)| \leq 0$. Lại thay $x=y=a$, ta có $f(0)=3 a \leq 0$. Lại thay $x=-a, y=a$ thì chú ý rằng $|-a|+a=0$, ta có $f(0)=3 a+|f(-a)|$ nên $f(-a)=0$, điều này kéo theo $a=-a$ hay $a=0$ (do tính duy nhất ở trên).

Thay $y=0$ thì $f(|x|)=|f(x)|$ nên $f(x) \geq 0, \forall x \geq 0$. Xét $x>0$ và $y=-\frac{f(x)}{3}$, ta có $f\left(x-\frac{f(x)}{3}+f\left(-\frac{f(x)}{3}+f\left(-\frac{f(x)}{3}\right)\right)\right)=0$ nên

$\quad\quad\quad\quad\quad\quad\quad\quad\quad\quad -\frac{f(x)}{3}+f\left(-\frac{f(x)}{3}+f\left(-\frac{f(x)}{3}\right)\right)=-x$

với mọi $x>0$. Trong đề bài, thay $x=0$ thì $f(y+f(y+f(y)))=3 y$. Thay $y \rightarrow-\frac{f(x)}{3}$ thì $f\left(-\frac{f(x)}{3}+f\left(-\frac{f(x)}{3}+f\left(-\frac{f(x)}{3}\right)\right)\right)=-f(x)$. So sánh hai đẳng thức trên, ta có $f(-x)=-f(x), \forall x>0$ nên $f$ là hàm số lẻ.

Từ tính chất hàm số lẻ, ta có $f\left(\frac{f(x)}{3}+f\left(\frac{f(x)}{3}+f\left(\frac{f(x)}{3}\right)\right)\right)=f(x)$ với mọi $x>0$. Trong đề bài, xét $x \geq 0$ và $y \rightarrow \frac{f(y)}{3}$, ta có

$\quad\quad\quad\quad\quad\quad\quad\quad\quad\quad f\left(x+\frac{f(y)}{3}+f\left(\frac{f(y)}{3}+f\left(\frac{f(y)}{3}\right)\right)\right)=f(y)+f(x)$

hay $f(x+y)=f(x)+f(y)$ với mọi $x, y>0$. Vì $f$ cộng tính trên $\mathbb{R}^{+}$nên ta có $f(x)=a x, \forall x>0$. Lại do tính chất hàm lẻ, ta suy ra $f(x)=a x, \forall x \in \mathbb{R}$. Thay vào đề bài, ta có $a=1$.

Vậy tất cả các hàm số cần tìm là $f(x)=x$.

Bài 3. Cho $p$ là số nguyên tố lẻ và $n=2 p+r$ với $r \in{0,1,2, \ldots, p-1}$. Đặt $X={1,2, \ldots, n}$. Ánh xạ $f: X \rightarrow X$ được gọi là có tính chất $\mathcal{P}$ nếu $f$ không phải là ánh xạ đồng nhất và $f(f(\ldots(f(k)) \ldots)$ ) $=k$ (ánh xạ hợp $p$ lần) với mọi $k \in X$.

Đặt $A_f={k \in X \mid f(k)=k}$.

a) Chứng minh rằng nếu $f$ có tính chất $\mathcal{P}$ thì $\left|A_f\right| \equiv r(\bmod p)$.

b) Gọi $d$ là số các ánh xạ có tính chất $\mathcal{P}$. Chứng minh rằng $d$ không là ước của $n$ !.

(Kí hiệu $|A|$ chỉ số lượng các phần tử của tập hợp $A$.)

Lời giải. a) Ta có

$\quad\quad\quad\quad\quad\quad\quad\quad\quad\quad \left|A_f\right| \equiv r \quad(\bmod p) \Leftrightarrow\left|X \backslash A_f\right| \text { chia hết cho } p \text {. }$

Điều này tương đương số phần tử của tập hợp $B={k \in X \mid f(k) \neq k}$ là bội của $p$. Đặt $f_m(x)$ là ánh xạ hợp $m$ lần. Xét $x \in B$ thì cũng có các số $f(x), f_2(x), \ldots, f_{p-1}(x) \in$ B. Thật vậy,

Giả sử tồn tại $1<m<p$ sao cho $f_m(x)=x$ với số $x \in B$ nào đó, ta chọn $m$ là số nhỏ nhất như thế. Vì $p$ nguyên tố lẻ nên $p$ không chia hết cho $m$. Do vậy tồn tại số $t$ sao cho $0<p-t m<m$. Lại có

$\quad\quad\quad\quad\quad\quad\quad\quad\quad\quad f_m(x)=x \Rightarrow f_{t m}(x)=x \Rightarrow f_{p-t m}(x)=f_p(x)=x$

(mâu thuẫn với tính nhỏ nhất của $m$ ). Vì thế nên với mọi $m$ mà $1<m<p$ thì $f_m(x) \neq x$. Từ đó suy ra với mọi $1<k<l<p$ thì $f_k(x) \neq f_l(x)$, tức là $x, f(x), f_2(x), \ldots, f_{p-1}(x)$ là $p$ số khác nhau thuộc $B$.

Xét số $y \in B$ và $y$ khác tất cả $p$ số ở trên. Khi đó, ta cũng sẽ có $y$ sinh ra một bộ $p$ số phân biệt mới. Giả sử rằng có $f_i(x)=f_j(y)$ với $i<j$ nào đó thì sẽ có $f_{p+i-j}(x)=f_p(y)=y$, mâu thuẫn. Suy ra trong $B$ sẽ có 1 hoặc 2 bộ $p$ số rời nhau, chứng tỏ rằng số phần tử của $B$ chia hết cho $p$. Suy ra điều phải chứng minh.

(b) Từ đây ta thấy rằng để đếm số ánh xạ $f$ có tính chất $\mathcal{P}$, trước hết, ta chọn ra $r$ hoặc $p+r$ vị trí cố định. Ta xét hai trường hợp như sau:

  1. Nếu $\left|A_f\right|=p+r$ thì có $C_n^{p+r}$ cách chọn ra các số này, còn lại $p$ số thì $f$ phải là song ánh trên tập con đó. Do đó trong trường hợp này có $p ! C_n^{p+r}$ cách.
  2. Nếu $\left|A_f\right|=r$ thì tương tự trên, ta cũng đếm được $(p !)^2 C_n^r C_{2 p}^p$.

Từ đó suy ra số ánh xạ tính chất $\mathcal{P}$ là

$\quad\quad\quad\quad\quad\quad\quad\quad\quad\quad d=p ! C_n^{p+r}+(p !)^2 C_n^r C_{2 p}^p$

Ta sẽ chứng minh số này không là ước của $n$ !. Ta viết số $d$ dưới dạng khai triển

$\quad\quad\quad\quad\quad\quad\quad\quad\quad\quad d=p ! \frac{n !}{(p+r) ! p !}+(p !)^2 \frac{n !}{r !(2 p) !} \cdot \frac{(2 p) !}{(p !)^2}=\frac{n !}{(p+r) !}+\frac{n !}{r !} .$

Đặt $(p+r) !=k \cdot(r !)^2$ với $k=\frac{(p+r) !}{(r !)^2}=\frac{p !}{r !} \cdot \frac{(p+r) !}{p ! r !}=\frac{p !}{r !} C_{p+r}^r \in \mathbb{Z}$. Khi đó, ta viết lại

$\quad\quad\quad\quad\quad\quad\quad\quad\quad\quad \frac{n !}{d}=\frac{r !(p+r) !}{r !+(p+r) !}=\frac{k \cdot(r !)^3}{(1+k \cdot r !) \cdot r !}=\frac{k \cdot(r !)^2}{k \cdot r !+1} .$

Dễ thấy số này không thể nguyên vì $k \cdot r !+1$ nguyên tố cùng nhau với $k \cdot(r !)^2$. Từ đó ta có $d$ không là ước của $n$ !.

Nhận xét. Bài này nếu tổng quát $n=k q+r$ thì kết quả câu a vẫn đúng. Tuy nhiên, câu b biến đổi sẽ phức tạp hơn nhiều.

Bài 4. Cho tam giác $A B C$ nội tiếp đường tròn $(O)$ có $A$ cố định và $B, C$ thay đổi trên $(O)$ sao cho $B C$ luôn song song với một đường thẳng cố định. Các tiếp tuyến của $(O)$ tại $B$ và $C$ cắt nhau tại $K$. Gọi $M$ là trung điểm của $B C, N$ là giao điểm của $A M$ với $(O)$. Chứng minh rằng đường thẳng $K N$ luôn đi qua một điểm cố định.

Lời giải. Giả sử $K N$ cắt $(O)$ tại $I$ thì tứ giác $B N C I$ điều hòa.

Do đó $A(B C, N I)=-1$, mà $A N$ chia đôi $B C$ nên $A I | B C$, tức là $A I$ có phương cố định. Từ đó ta thấy $I$ là điểm cố định cần tìm.

Ngày thi thứ hai

Bài 5. Chứng minh rằng nếu $a, b, c$ là độ dài ba cạnh của một tam giác thì:

$\quad\quad\quad\quad\quad\quad\quad\quad\quad\quad (2 a+2 b-c)(2 b+2 c-a)(2 c+2 a-b)>25 a b c .$

Lời giải. Đặt $a+b-c=x, b+c-a=y, c+a-b=z$ thì $x, y, z>0$. Ta đưa về bất đẳng thức

$\quad\quad\quad\quad\quad\quad\quad\quad\quad\quad \left(4 \cdot \frac{x}{y+z}+1\right)\left(4 \cdot \frac{y}{z+x}+1\right)\left(4 \cdot \frac{z}{x+y}+1\right)>25 .$

Không mất tính tổng quát, giả sử $0<x \leq y \leq z$. Đặt $S=x+y+z$. Ta đưa về

$\quad\quad\quad\quad\quad\quad\quad\quad\quad (S+3 x)(S+3 y)(S+3 z)>25(S-x)(S-y)(S-z) .$

Khai triển và rút gọn, ta được

$\quad\quad\quad\quad\quad\quad\quad\quad\quad\quad\quad\quad S^3-4 S(x y+y z+z x)+13 x y z>0 .$

Chú ý rằng

$\quad\quad\quad\quad S^3-4 S(x y+y z+z x)=S\left(S^2-4(x y+y z+z x)\right)=S\left((x+y-z)^2-4 x y\right)$

nên ta đưa về $S(x+y-z)^2+x y(13 z-4 S)>0$. Bất đẳng thức cuối đúng vì $13 c-4 S=9 z-4(x+y)>0$.

Bài 6. Cho dãy số $\left(u_n\right)$ thoả mãn $u_1=\sqrt{2}$ và $u_{n+1}=\frac{2 u_n^2+5 u_n+5}{2 u_n+4}, \forall n \geq 1$. Tìm $\lim \frac{u_n^2-3 u_n+5}{3 n^2+4 n-1}$.

Lời giải. Ta thấy rằng $u_n>0, \forall n$ và $u_{n+1}-u_n=\frac{u_n+5}{2\left(u_n+2\right)}>0$ nên dãy tăng. Giả sử dãy bị chặn trên thì nó hội tụ về $L>0$, suy ra

$\quad\quad\quad\quad\quad\quad\quad\quad\quad L=\frac{2 L^2+5 L+5}{2 L+4} \Leftrightarrow L=-5,$

vô lý. Suy ra $\lim _{n \rightarrow+\infty} u_n=+\infty$. Từ đó, ta được

nên theo định lý Stolz, ta suy ra $\lim _{n \rightarrow+\infty} \frac{u_n}{n}=\frac{1}{2}$ và $\lim _{n \rightarrow+\infty} \frac{u_n}{n^2}=0$. Do đó, trong biểu thức cần tính giới hạn, chia tử và mẫu cho $n^2$ rồi áp dụng kết quả trên, ta có

$\quad\quad\quad\quad\quad \lim _{n \rightarrow+\infty} \frac{u_n^2-3 u_n+5}{3 n^2+4 n-1}=\lim _{n \rightarrow+\infty} \frac{\left(\frac{u_n}{n}\right)^2-\frac{3 u_n-5}{n^2}}{3+\frac{4}{n}-\frac{1}{n^2}}=\left(\frac{1}{2}\right)^2 \cdot \frac{1}{3}=\frac{1}{12}$

Bài 7. Xét số tự nhiên $n>1$. Bắt đầu từ bộ số $1,2, \ldots, 2 n-1,2 n$, ta thực hiện phép biến đổi sau: Chọn hai số $a, b$ sao cho $a-b>1$, xoá hai số này và thay thế bởi hai số $a-1, b+1$. Với bộ số mới, ta lại tiếp tục thực hiện phép biến đổi tương tự.

a) Chứng minh rằng ta sẽ đạt đến trạng thái dừng, tức là không thể tiếp tục thực hiện phép biến đổi như vậy được nữa.

b) Gọi $k$ là số lần phép biến đổi cần thực hiện để đạt đến trạng thái dừng. Tìm giá trị nhỏ nhất và lớn nhất của $k$.

Lời giải. (a) Xét đại lượng $S$ là tổng bình phương các số thu được sau mỗi thao tác biến đổi.

Ta thấy rằng từ $(a, b)$ với $a-b>1$, ta đưa về bộ $(a-1, b+1)$ thì tổng trên thay đổi một lượng là $a^2+b^2-(a-1)^2-(b+1)^2=2(a+b-1)>0$. Do đó, tổng $S$ giảm ngặt, và rõ ràng $S$ phải luôn dương nên thao tác trên chỉ thực hiện được trong hữu hạn lần.

(b) Rõ ràng tổng trên không đổi khi không còn cặp số $a, b$ nào mà $a-b>1$. Điều này đồng nghĩa với việc các số thu được trong trạng thái cuối chỉ nhận hai giá trị liên tiếp nào đó. Ta thấy rằng tổng các số đã cho luôn không đổi và là $1+2+\cdots+2 n=n(2 n+1)$

Giả sử cuối cùng, ta có $x$ số $m$ và $y$ số $m+1$ thì

$\quad\quad\quad\quad\quad\quad\quad\quad\quad\quad\quad\quad\left\{\begin{array}{l}x+y=2 n \\ m x+(m+1) y=n(2 n+1)\end{array}\right.$

Suy ra $2 m n+y=2 n^2+n \Rightarrow n \mid y$. Tuy nhiên, nếu $y \in{0,2 n}$ thì vô lý vì vế phải không chia hết cho $2 n$. Do đó $x=y=n$ và $m=n$, tức là ở trạng thái cuối, ta còn $n$ số $n$ và $n+1$.

  • Tổng bình phương của chúng là $S=n \cdot n^2+n \cdot(n+1)^2=n\left(2 n^2+2 n+1\right)$.
  • Tổng bình phương ban đầu là $S_0=1^2+2^2+\cdots+(2 n)^2=\frac{n(2 n+1)(4 n+1)}{3}$.

Suy ra $S_0-S=\frac{2}{3}\left(n^3-n\right)$.

(b) Để thực hiện được nhiều lần nhất thì giá trị giảm đi ở mỗi lần phải ít nhất. Theo câu a) thì giá trị đó sẽ là $2(a+b-1) \geq 2$.

Suy ra số lần nhiều nhất sẽ là $\frac{1}{3}\left(n^3-n\right)$. Để thực hiện được điều này, ta sẽ cố gắng trong mỗi thao tác tạo ra nhiều giá trị nhất có thể và đồng thời làm giảm số lượng các giá trị ở hai biên đi. Từ đó ta được $k_{\max }=\frac{1}{3}\left(n^3-n\right)$.

Để thực hiện được ít lần nhất, ta sử dụng ý tưởng tham lam, mỗi lần, ta sẽ chọn các cặp số nằm về hai phía của $n, n+1$. Khi đó, giá trị của các số $1,2, \ldots, n-1$ sẽ dần dần được tăng lên, trong khi giá trị của các số $n+2, n+3, \ldots, 2 n$ dần dần sẽ giảm đi. Tổng khoảng cách từ các số nhỏ hơn $n$ đến $n$ là $1+2+\cdots+n-1=\frac{n(n-1)}{2}$. Tương tự thì tổng khoảng cách các số lớn hơn $n+1$ đến $n+1$ cũng là $\frac{n(n-1)}{2}$. Ta thấy mỗi lần thao tác thì các số này sẽ thu hẹp khoảng cách đúng 2 đơn vị nên số lần thao tác tối thiểu phải là $\frac{1}{2}\left(\frac{n(n-1)}{2}+\frac{n(n-1)}{2}\right)=\frac{n(n-1)}{2}$.

Để đạt được giá trị này, mỗi lần, ta chỉ cần chọn các cặp số có dạng $(t, 2 n+1-t)$ với $1 \leq t \leq n-1$ là được. Suy ra $k_{\min }=\frac{n(n-1)}{2}$.

Bài 8. Cho đường tròn $\left(\gamma_1\right)$ đường kính $A B$ và đường tròn $\left(\gamma_2\right)$ tâm $A$ cắt $\left(\gamma_1\right)$ tại $C, D$. Điểm $M$ thay đổi trên cung $C D$ (nằm bên trong $\left(\gamma_1\right)$ ) của $\left(\gamma_2\right)$. Gọi $B M$ cắt $\left(\gamma_1\right)$ tại $N$ khác $M$ và $B$. Tìm giá trị nhỏ nhất của $\frac{N D+N C}{M N}$.

Lời giải. Theo định lý Ptolemy cho tứ giác $B C N D$ nội tiếp trong $\gamma_1$ thì

$\quad\quad\quad\quad\quad\quad\quad\quad\quad\quad\quad B C \cdot N D+B D \cdot N C=B N \cdot C D .$

Vì $A C=A D$ nên $B C=B D=m$ và $C D=n$ là các giá trị cố định.

Ta có

$\quad\quad\quad\quad\quad\quad\quad\quad m(N C+N D)=n \cdot B N \Rightarrow N C+N D=\frac{n}{m} \cdot B N .$

Suy ra $\frac{N C+N D}{M N}=\frac{n}{m} \cdot \frac{B N}{M N}$. Ta đưa về tìm giá trị nhỏ nhất của $\frac{B N}{M N}$. Xét phương tích từ $B$ đến $\gamma_2$ thì $B M \cdot B N=B K \cdot B A=c$ là hằng số nên$(B N-M N) B N=c$. Do đó $\frac{M N}{B N}=1-\frac{c}{B N^2}$ nên

$\quad\quad\quad\quad\quad\quad\frac{B N}{M N} \min \Leftrightarrow \frac{M N}{B N} \max \Leftrightarrow \frac{c}{B N^2} \min \Leftrightarrow B N^2 \max .$

Dễ thấy $\max B N=A B$, xảy ra khi $N \equiv A$ hay $M \equiv K$. Khi đó

$\quad\quad\quad\quad\quad\quad\quad\quad\quad\quad\quad\frac{N C+N D}{M N}=\frac{A C+A D}{A K}=2$

chính là giá trị nhỏ nhất cần tìm.

 

 

 

 

 

 

 

 

 

 

 

 

 

 

 

 

 

 

 

 

 

 

 

 

 

 

 

 

 

 

 

 

 

 

 

 

 

 

 

 

 

 

 

 

 

 

 

 

 

 

 

 

 

 

 

 

 

 

 

 

 

 

 

 

 

 

 

 

 

 

 

 

 

 

 

 

 

 

 

 

 

 

 

 

 

 

 

 

 

 

 

 

 

 

 

 

 

Đề thi và đáp án kì thi chọn đội tuyển thi Quốc gia trường Phổ thông Năng khiếu năm học 2009 – 2010

ĐỀ THI

Ngày thi thứ nhất

Bài 1. Cho $a, b, c$ là các số thực để đa thức $P(x)=x^4+ax^3+b x^2+c x+1$ có ít nhất một nghiệm thực. Tìm tất cả các bộ $(a, b, c)$ để $a^2+b^2+c^2$ đạt giá trị nhỏ nhất.

Bài 2. Cho $A={1,2, \ldots, 2 n}$. Một tập con của $A$ được gọi là tốt nếu như có đúng 2 phần tử $x, y$ và đồng thời $|x-y| \in[1, n]$. Tìm số các tập hợp $[A_1, A_2, \ldots, A_n]$ để $A_i$ là tập con tốt của $A$ với $1 \leq i \leq n$ và $\bigcup_{i=1}^n A_i=A$.

Bài 3. Tìm tất cả các hàm số $f: \mathbb{N}^* \rightarrow \mathbb{N}^*$ thoả mãn các điều kiện sau:

$\quad\quad(i) f $ là hàm số tăng thật sự trên $\mathbb{N}^*$.

$\quad\quad(ii) f(f(n))=4 n+9 \forall n \in \mathbb{N}^*$.

$\quad\quad(iii) f(f(n)-n)=2 n+9 \forall n \in \mathbb{N}^*$.

Bài 4. Cho đường tròn tâm $O$ và dây cung $A B$ cố định khác đường kính. Một điểm $P$ thay đổi trên cung lớn $A B$. Gọi $I$ là trung điểm của $A B$. Lấy các điểm $M, N$ trên các tia $P A, P B$ sao cho $\angle P M I=\angle P N I=\angle A P B$.

(a) Chứng minh rằng đường cao từ $P$ của tam giác $P M N$ luôn đi qua một điểm cố định.

(b) Chứng minh rằng đường thẳng Euler của tam giác $P M N$ luôn đi qua một điểm cố định.

Ngày thi thứ hai

Bài 5. Cho $a, b, c$ là các số thực dương. Giải hệ phương trình sau:

$\quad\quad\quad\quad\quad\quad\quad\quad\quad\quad\left\{\begin{array}{l}a x-a b y+\frac{1}{x y}=b c^2 \\ a b z-b c^2 x+\frac{1}{x z}=a . \\ b c^2 y-a z+\frac{1}{y z}=a b\end{array}\right.$

Bài 6. Cho dãy số $\left(a_n\right)$ xác định bởi $a_1=a, a_{n+1}=\left(a_1+\cdots+a_n-2\right)^2 \forall n \in \mathbb{N}^*$. Đặt $S_n=a_1+a_2+\cdots+a_n$. Tìm tất cả các giá trị $a$ để dãy số $\left(S_n\right)$ hội tụ.

Bài 7. Tìm tất cả các số nguyên dương $k$ để phương trình sau có nghiệm nguyên dương $(x, y)$ :

$$\quad\quad x^2+y^2+x+y=k x y$

Bài 8. Cho tam giác $A B C$ nội tiếp đường tròn $(O)$. Gọi $I, I_1, I_2, I_3$ lần lượt là tâm đường tròn nội tiếp và tâm đường tròn bàng tiếp các đỉnh $A, B, C$ của tam giác $A B C$. Dường tròn ngoại tiếp tam giác $I I_2 I_3$ cắt $(O)$ tại hai điểm $M_1, N_1$. Gọi $J_1$ là giao điểm của $A I$ và $(O)$. Ký hiệu $d_1$ là đường thẳng qua $J_1$ và vuông góc với $M_1 N_1$. Xác định các đường thẳng $d_2, d_3$ tương tự. Chứng minh rằng $d_1, d_2, d_3$ dồng quy.

 

LỜI GIẢI

Ngày thi thứ nhất

Bài 1. Cho $a, b, c$ là các số thực để đa thức $P(x)=x^4+a x+3+b x^2+c x+1$ có ít nhất một nghiệm thực. Tìm tất cả các bộ $(a, b, c)$ để $a^2+b^2+c^2$ đạt giá trị nhỏ nhất.

Lời giải. Gọi $x_0$ là một nghiệm của $P(x)$ (dễ thấy $x_0 \neq 0$ ). Do $P\left(x_0\right)=0$ nên ta có

$\quad\quad\quad\quad\quad\quad\quad\quad\quad\quad-\left(x_0^4+1\right)=a x_0^3+b x_0^2+c x_0 .$

Sử dụng bất đẳng thức Cauchy-Schwarz, ta có

$\quad\quad\quad\quad\quad\left(x_0^4+1\right)^2=\left(a x_0^3+b x_0^2+c x_0\right)^2 \leq\left(a^2+b^2+c^2\right)\left(x_0^6+x_0^4+x_0^2\right) .$

Đặt $t=x_0^2>0$. Từ đánh giá trên, ta suy ra

$\quad\quad\quad\quad\quad\quad\quad\quad\quad\quad a^2+b^2+c^2 \geq \frac{\left(t^2+1\right)^2}{t^3+t^2+t}=\frac{\left(t^2+1\right)^2}{t\left(t^2+t+1\right)}$

Mà theo bất đẳng thức AM-GM thì

$\quad\quad\quad\quad\quad\quad\quad\quad\quad\quad t \leq \frac{t^2+1}{2} \text { và } t^2+t+1 \leq t^2+\frac{t^2+1}{2}=\frac{3}{2}\left(t^2+1\right) .$

Do đó

$\quad\quad\quad\quad\quad\quad\quad\quad\quad\quad\frac{\left(t^2+1\right)^2}{t\left(t^2+t+1\right)} \geq \frac{4}{3}, \text { nên } a^2+b^2+c^2 \geq \frac{4}{3}$

Đẳng thức xảy ra khi và chỉ khi

$\quad\quad\quad\quad\quad\quad\quad\quad\quad\quad\left\{\begin{array}{l}x_0^4+a x_0^3+b x_0^2+c x_0+1=0 \\ x_0^2=1 \\ \frac{a}{x_0^3}=\frac{b}{x_0^2}=\frac{c}{x_0}\end{array}\right.$

Giải hệ này, ta thu được $a=b=c=-\frac{2}{3}$ hoặc $a=-b=c=\frac{2}{3}$.

Bài 2. Cho $A=[1,2, \ldots, 2 n]$. Một tập con của $A$ được gọi là tốt nếu như có đúng 2 phần tử $x, y$ và đồng thời $|x-y| \in[1, n]$. Tìm số các tập hợp $[A_1, A_2, \ldots, A_n]$ để $A_i$ là tập con tốt của $A$ với $1 \leq i \leq n$ và $\bigcup_{i=1}^n A_i=A$.

Lời giải . Gọi $u_n, n \in{1,2, \ldots, n}$ là số các tập hợp $[A_1, A_2, \ldots, A_n]$ thỏa mãn yêu cầu đề bài, đồng thời hai phần tử $n$ và $n+1$ không đi cùng nhau trong bất kì tập $A_i$ nào. Ta chia các số $1,2, \ldots, 2 n$ vào một bảng $2 \times n$ như sau

$\quad\quad\quad\quad\quad\quad\quad\quad\quad\quad\quad\quad\quad\quad\begin{array}{|c|c|c|c|}\hline 1 & 2 & \ldots & n \\ \hline n+1 & n+2 & \ldots & 2 n \\ \hline\end{array}$

Khi đó, mỗi cách chọn được liệt kê trong $u_n$ tương ứng với một cách chọn từ bảng trên các cặp gồm hai số ở cùng một cột hoặc hai số liên tiếp nhau trên cùng một hàng. Xét $u_{n+1}$, vì phần tử $2(n+1)$ chỉ có thể đi cùng với $n+1$ hoặc $2 n+1$ trong cùng một tập $A_i$ nào đó nên ta xét hai khả năng sau.

  • $2(n+1)$ và $n+1$ cùng thuộc một tập $A_i$, giả sử là $A_{n+1}$.

$\quad\quad\quad\quad\quad\quad\quad\quad\quad\quad\quad\quad\begin{array}{|c|c|l|c|}\hline 1 & 2 & \ldots & n+1 \\ \hline n+1 & n+2 & \ldots & 2(n+1) \\ \hline\end{array}$

Lúc này, mỗi cách chọn một bộ $[A_1, A_2, \ldots, A_n]$ ứng với một cách chọn các cặp số gồm các số ở cùng một cột hoặc ở cạnh nhau trong cùng một hàng từ một bảng $2 \times n$. Theo định nghĩa của ta số cách chọn như thế là $u_n$. Vậy trong trường hợp này có $u_n$ cách chọn.

  • $2(n+1)$ và $2 n+1$ cùng thuộc một tập $A_i$, giả sử đó là $A_{n+1}$.

$\quad\quad\quad\quad\quad\quad\quad\quad\quad\quad\quad\quad\begin{array}{|c|c|l|c|c|}\hline 1 & 2 & \ldots & n & n+1 \\ \hline n+1 & n+2 & \ldots & 2 n+1 & 2(n+1) \\ \hline\end{array}$

Ta thấy $n+1$ chỉ có thể đi cùng với $2(n+1)$ (trường hợp $n+1$ đi cùng với $n+2$ không được xét trong $\left.u_{n+1}\right)$ nhưng $2(n+1)$ đã đi cùng với $2 n+1$ nên $n+1$ phải đi cùng với $n$ trong cùng một tập $A_i$ nào đó, giả sử là $A_n$. Lập luận tương tự trường hợp trên, ta suy ra số cách chọn các tập ${A_1, A_2, \ldots, A_n-1}$ là $u_{n-1}$.

Theo quy tắc cộng, ta có $u_{n+1}=u_n+u_{n-1}$. Mặt khác, $u_1=1$ và $u_2=2$ nên ta tìm được công thức tổng quát của $u_n$ là

$\quad\quad\quad\quad\quad\quad\quad\quad\quad\quad\quad\quad u_n=\frac{1}{\sqrt{5}}\left[\left(\frac{1+\sqrt{5}}{2}\right)^{n+1}-\left(\frac{1-\sqrt{5}}{2}\right)^{n+1}\right]$

Xét trường hợp sinh ra bộ $[A_1, A_2, \ldots, A_n]$ có $n$ và $n+1$ đi cùng trong một tập $A_i$ nào đó.

$\quad\quad\quad\quad\quad\quad\quad\quad\quad\quad\quad\quad\begin{array}{|c|c|l|c|}\hline 1 & 2 & \ldots & n \\ \hline n+1 & n+2 & \ldots & 2 n \\ \hline\end{array}$

Rõ ràng 1 chỉ có thể đi cùng với 2 hoặc $n+1$ nhưng $n+1$ đã đi cùng $n$ nên 1 chỉ có thể đi cùng với 2 . Tiếp theo, $n+2$ có thể đi cùng với $2, n+1$ hay $n+3$ nhưng 2 đã đi với 1 còn $n+1$ đã đi với $n$ nên $n+2$ phải đi với $n+3$.

Tiếp tục, 3 có thể đi cùng 2,4 hay $n+3$ nhưng 2 đã đi với 1 còn $n+3$ đã đi với $n+2$ nên 3 phải đi với 4 . Tiếp tục lý luận như trên, ta suy ra $A_i$ phải có dạng

$\quad\quad\quad [1,2],[3,4], \ldots,[n-2, n-1],[n, n+1],[n+1, n+2], \ldots,[2 n-1,2 n]$

Từ đó suy ra trường hợp này chỉ cho ta duy nhất một bộ $[A_1, A_2, \ldots A_n]$ nếu $n$ lẻ và không có bộ nào nếu $n$ chẵn.

Vậy số các bộ $[A_1, A_2, \ldots A_n]$ thỏa mãn đề bài là

$\quad\quad\quad\quad\quad\quad\quad\quad\begin{cases}\frac{1}{\sqrt{5}}\left[\left(\frac{1+\sqrt{5}}{2}\right)^{n+1}-\left(\frac{1-\sqrt{5}}{2}\right)^{n+1}\right] & , n=1,n \text { chẳn. } \\ 1+\frac{1}{\sqrt{5}}\left[\left(\frac{1+\sqrt{5}}{2}\right)^{n+1}-\left(\frac{1-\sqrt{5}}{2}\right)^{n+1}\right] & , n>1, n \text { lẻ’ }\end{cases}$

Bài 3. Tìm tất cả các hàm số $f: \mathbb{N}^* \rightarrow \mathbb{N}^*$ thoả mãn các điều kiện sau:

$\quad\quad(i) f $ là hàm số tăng thật sự trên $\mathbb{N}^*$.

$\quad\quad(ii) f(f(n))=4 n+9 \forall n \in \mathbb{N}^*$.

$\quad\quad(iii) f(f(n)-n)=2 n+9 \forall n \in \mathbb{N}^*$.

Lời giải. Vì $f: \mathbb{N}^* \longrightarrow \mathbb{N}^*$ tăng ngặt nên

$\quad\quad\quad\quad\quad\quad\quad\quad\quad\quad f(a)-f(b) \geq a-b, \forall a, b \in \mathbb{N}^*, a>b .$

Theo điều kiện (iii), ta có

$2=2(n+1)+9-(2 n+9) =f(f(n+1)-(n+1))-f(f(n)-n) $

$\quad\quad\quad\quad\quad\quad\quad\quad\quad\quad\quad\quad \geq f(n+1)-(n+1)-[f(n)-n] $

$\quad\quad\quad\quad\quad\quad\quad\quad\quad\quad\quad\quad =f(n+1)-f(n)-1 .$

Do đó $f(n+1)-f(n) \leq 3$

với mọi $n \in \mathbb{N}^*$, tức $f(n+1)-f(n) \in\ {1,2,3}$

với mọi $n \in \mathbb{N}^*$. Ta xét các trường hợp sau

  • Giả sử tồn tại $n \in \mathbb{N}^*$ sao cho $f(n+1)-f(n)=1$ thì

$\quad\quad\quad\quad\quad\quad\quad\quad\quad\quad f(n+1)-(n+1)=f(n)-n,$

suy ra $2(n+1)+9=f(f(n+1)-(n+1))=f(f(n)-n))=2 n+9$, vô lí.

  • Giả sử tồn tại $n \in \mathbb{N}^*$ sao cho $f(n+1)-f(n)=3$ thì

$\quad\quad\quad\quad\quad\quad\quad\quad\quad\quad f(n+1)-(n+1)=f(n)-n+2$

Ta lại có

$\quad\quad\quad\quad\quad\quad\quad\quad\quad\quad f(f(n+1)-(n+1))-f(f(n)-n)=2$

Do đó, nếu $f(n)>n$ thì đặt $t=f(n)-n \in \mathbb{N}^*$, ta suy ra $f(t+2)=t+2$.

Mà $f$ tăng ngặt nên $f(t+1)-f(t)=1$, mâu thuẫn.

Vậy $f(n) \leq n$ với mọi $n \in \mathbb{N}^*$ suy ra

$f(n)=n$ với mọi $n \in \mathbb{N}^*$.

Khi đó ta lại có $n=f(n)=f(f(n))=4 n+9,$ tức $n=-3$, vô lí.

Như vậy, $f(n+1)-f(n) \notin{1,3}$ với mọi $n \in \mathbb{N}^*$, nên

$\quad\quad\quad\quad\quad\quad\quad\quad\quad\quad f(n+1)-f(n)=2 \text {, với mọi } n \in \mathbb{N}^* \text {. }$

Ta suy ra $f(n)=2 n+k$. Thay vào đề bài, ta thu được $k=3$. Vậy $f(n)=2 n+3$ là nghiệm duy nhất của phương trình.

Bài 4. Cho đường tròn tâm $O$ và dây cung $A B$ cố định khác đường kính. Một điểm $P$ thay đổi trên cung lớn $A B$. Gọi $I$ là trung điểm của $A B$. Lấy các điểm $M, N$ trên các tia $P A, P B$ sao cho $\angle P M I=\angle P N I=\angle A P B$.

(a) Chứng minh rằng đường cao từ $P$ của tam giác $P M N$ luôn đi qua một điểm cố định.

(b) Chứng minh rằng đường thẳng Euler của tam giác $P M N$ luôn đi qua một điểm cố định.

Lời giải. (a) Kí hiệu $X=M I \cap P B, Y=N I \cap P A$. Ta có $\angle P M I=\angle P N I=$ $\angle A P B$ nên các tam giác $P M X$ và $P N Y$ cân tại $X, Y$. Từ đó suy ra

$\angle P X M=\angle P Y N=180^{\circ}-2 \angle A P B,$

suy ra $M, N, X, Y$ đồng viên. Gọi $S$ là tâm đường tròn ngoại tiếp tam giác $A O B$ thì $S$ cố định. Ta có $\angle I S B=180^{\circ}-\angle A O B=180^{\circ}-2 \angle A P B=\angle P X M$. Tương tự, ta suy ra $\angle I S A=\angle P Y N$. Do đó $I, S, X, B$ đồng viên và $I, S, Y, A$ đồng viên. Suy ra

$\angle S X B=\angle S Y A=\angle S I B=90^{\circ} .$

Suy ra $I S$ là đường kính của đường tròn ngoại tiếp tam giác $P X Y$. Mặt khác, $M, N, X, Y$ dồng viên nên nên $M N$ và $X Y$ đối song nhau trong $\angle A P B$, tức $I S \perp$ $M N$. Nói cách khác, đường cao từ $P$ của tam giác $P M N$ đi qua điểm $S$ cố định.

(b) Trước tiên, ta chứng minh bổ đề sau.

BỔ ĐỀ. Cho tam giác $A B C$ và đường tròn $(\omega)$ đi qua hai điểm $B, C$ và cắt các canh $A B, A C$ tại $X, Y$. Gọi $X X^{\prime}, Y Y^{\prime}$ là các đường cao của tam giác $A X Y$. Gọi $B B^{\prime}, C C^{\prime}$ là các đường cao của tam giác $A B C$. Gọi $H, H^{\prime}$ là các trục tâm của tam giác $A B C$ và tam giác $A X Y$. Kí hiệu $I \equiv B Y \cap C X$. Khi đó $H, I, H^{\prime}$ thẳng hàng.

Chứng minh. Ta có $X, Y, X^{\prime}, Y^{\prime}$ đồng viên nên $\overline{H^{\prime} X} \cdot \overline{H^{\prime} X^{\prime}}=\overline{H^{\prime} Y} \cdot \overline{H^{\prime} Y^{\prime}}$, tức là

$P_{H^{\prime} /[B Y]}=P_{H^{\prime} /[C X]},$

trong đó $[U V]$ là đường tròn đường kính $U V$. Ta có $B, C, B^{\prime}, C^{\prime}$ đồng viên nên $\overline{H B} \cdot \overline{H B^{\prime}}=\overline{H C} \cdot \overline{H C^{\prime}}$, tức

$P_{H /[B Y]}=P_{H /[C X]} .$

Cuối cùng $B, C, X, Y$ đồng viên nên $\overline{I B} \cdot \overline{I Y}=\overline{I C} \cdot \overline{I X}$, tức

$P_{I /[B Y]}=P_{I /[C Y]} .$

Suy ra $H, I, H^{\prime}$ thẳng hàng vì cùng thuộc trục đẳng phương của $[B Y]$ và $[C X]$.

Trở lại bài toán,

Gọi $H, O^{\prime}$ lần lượt là trực tâm và tâm đường tròn ngoại tiếp của tam giác $P M N$. Ta có $O^{\prime} P=O^{\prime} M$ và $X P=X M$ nên $X O^{\prime}$ là đường trung trực của $P M$, suy ra $X O^{\prime} \perp P Y$. Tương tự ta cũng có $Y O^{\prime} \perp P X$.

Vì thế nên $O^{\prime}$ cũng chính là trực tâm của tam giác $P X Y$. Áp dụng bổ đề cho tam giác $P X Y$ với $(\omega) \equiv(M N X Y)$ thì ta có $O^{\prime}, H, I \equiv Y N \cap M X$ thẳng hàng. Hay nói cách khác, đường thẳng Euler $O^{\prime} H$ của tam giác $P M N$ đi qua điểm $I$ cố định. Bài toán được giải quyết.

Ngày thi thứ hai

Bài 5. Cho $a, b, c$ là các số thực dương. Giải hệ phương trình sau:

$\quad\quad\quad\quad\quad\quad\quad\quad\quad\quad\left\{\begin{array}{l}a x-a b y+\frac{1}{x y}=b c^2 \\ a b z-b c^2 x+\frac{1}{x z}=a \\ b c^2 y-a z+\frac{1}{y z}=a b\end{array}\right.$

Lời giải. Đặt $(m, n, p)=\left(a, a b, b c^2\right)$. Khi đó $m, n, p>0$. Hệ phương trình trở thành

$\quad\quad\quad\quad\quad\quad\quad\quad\quad\quad\left\{\begin{array}{l}m x-n y+\frac{1}{x y}=p \\ n z-p x+\frac{1}{z x}=m \\ p y-m z+\frac{1}{y z}=n,\end{array}\right.$

tương đương

$\quad\quad\quad\quad\quad\quad\quad\quad\quad\quad\left\{\begin{aligned}m x-n y-p &=-\frac{1}{x y}, \\ -m+n z-p x &=-\frac{1}{z x}, \\ -m z+p y-n &=-\frac{1}{y z} .\end{aligned}\right.$

Xem hệ trên là hệ phương trình tuyến tính theo ẩn $m, n, p$, ta có

$\quad\quad D=\left|\begin{array}{ccc}x & -y & -1 \\ -1 & z & -x \\ -z & -1 & y\end{array}\right|=x y z-x y z-1-x^2-y^2-z^2=-1-\left(x^2+y^2+z^2\right) \neq 0$

$\quad\quad\quad\quad\quad\quad\quad \quad\quad D_m=\left|\begin{array}{ccc}x & -\frac{1}{x y} & -1 \\ -1 & -\frac{1}{z x} & -x \\ -z & -\frac{1}{y z} & y\end{array}\right|=-\frac{1+x^2+y^2+z^2}{z x} .$

Tương tự, ta cũng tính được

$\quad\quad\quad\quad\quad\quad D_n=-\frac{1+x^2+y^2+z^2}{y z} \text {, và } D_p=-\frac{1+x^2+y^2+z^2}{x y} .$

Do đó

$\quad\quad\quad\quad\quad\quad\quad (m, n, p)=\left(\frac{D_m}{D}, \frac{D_n}{D}, \frac{D_p}{D}\right)=\left(\frac{1}{z x}, \frac{1}{y z}, \frac{1}{x y}\right) .$

Thay $(m, n, p)=\left(a, a b, b c^2\right)$, ta được

$\quad\quad\quad\quad\quad\quad\quad\quad\quad\quad x y=\frac{1}{b c^2}, y z=\frac{1}{a b}, z x=\frac{1}{a} .$

Nhân ba phương trình trên vế theo vế rồi lấy căn bậc hai, ta được $x y z=\pm \frac{1}{a b c}$.

  • Với $x y z=\frac{1}{a b c}$, ta suy ra $x=\frac{1}{c}, y=\frac{1}{b c}, z=\frac{c}{a}$.
  • Với $x y z=-\frac{1}{a b c}$, ta suy ra $x=-\frac{1}{c}, y=-\frac{1}{b c}, z=-\frac{c}{a}$.

Vậy hệ có 2 nghiệm là $\left(\frac{1}{c}, \frac{1}{b c}, \frac{c}{a}\right)$ và $\left(-\frac{1}{c},-\frac{1}{b c},-\frac{c}{a}\right)$.

Bài 6. Cho dãy số $\left(a_n\right)$ xác định bởi $a_1=a, a_{n+1}=\left(a_1+\cdots+a_n-2\right)^2 \forall n \in \mathbb{N}^*$.

Đặt $S_n=a_1+a_2+\cdots+a_n$. Tìm tất cả các giá trị $a$ để dãy số $\left(S_n\right)$ hội tụ.

Lời giải. Từ giả thiết suy ra $S_{n+1}-S_n=\left(S_n-2\right)^2$. Do đó dãy $\left(S_n\right)$ được xác định như sau

$\quad\quad\quad\quad\quad\quad\quad\quad\quad\quad\left\{\begin{array}{l}S_1=a, \\ S_{n+1}=f\left(S_n\right)=S_n^2-3 S_n+4 .\end{array}\right.$

Hơn nữa $f^{\prime}(x)=0$ có nghiệm duy nhất $x=\frac{2}{3}$ nên ta có thể vẽ bảng biến thiên, khảo sát được hàm số này. Từ đó, nhờ việc các trường hợp của $a$, ta thấy

  • Nếu $a>2$. Giả sử $[S_n]$ có giới hạn $L$ thì ta phải có $L=f(L)$ nên $L \in{1,2}$. Mà $\left(S_n\right)$ không giảm nên $L \geq a>2$, mâu thuẫn. Vậy nếu $a>2$ thì $\left(S_n\right)$ không hội tụ.
  • Nếu $a<1$ thì suy ra $S_2=f(a)>2$. Quay về trường hợp 1 , ta suy ra $\left(S_n\right)$ không hội tụ.
  • Nếu $1 \leq a \leq 2$ thì từ bảng biến thiên, ta có $\frac{7}{4} \leq S_n \leq 2$ với mọi $n \in \mathbb{N}^*$. Từ đó $\left(S_n\right)$ không giảm và bị chặn nên $\left(S_n\right)$ hội tụ.

Vậy các giá trị của $a$ thỏa mãn đề bài là $a \in[1,2]$.

Bài 7. Tìm tất cả các số nguyên dương $k$ để phương trình sau có nghiệm nguyên dương $(x, y)$ :

$\quad\quad\quad\quad\quad\quad\quad\quad\quad\quad\quad x^2+y^2+x+y=k x y .$

Lời giải. Không mất tính tổng quát, giả sử $x \geq y$. Xét giá trị $k$ sao cho phương trình đã cho có nghiệm nguyên dương. Trong các nghiệm ấy, gọi $\left(x_0, y_0\right)$ là nghiệm sao cho $x_0 \geq y_0 \geq 0$ và $x_0$ nhỏ nhất. Xét tam thức

$\quad\quad\quad\quad\quad\quad\quad\quad\quad f(x)=x^2-\left(k y_0-1\right) x+y_0^2+y_0 .$

Khi đó $f\left(x_0\right)=0$. Theo định lí Viette, $f(x)$ còn một nghiệm khác là $x_0^{\prime}=k y_0-1-x_0$. Tuy nhiên, theo cách chọn $\left(x_0, y_0\right)$ thì ta có $x_0^{\prime} \geq x_0 \geq y_0$ nên $y_0$ nằm ngoài hai khoảng nghiệm của tam thức bậc hai $f(x)$. Mà hệ số cao nhất của $f(x)$ dương nên $f\left(y_0\right) \geq 0$. Do $f\left(y_0\right) \geq 2 y_0^2+2 y_0-k y_0^2$ nên ta có

$\quad\quad\quad\quad\quad\quad\quad\quad\quad\quad\quad k \leq 2+\frac{2}{y_0} \leq 4 .$

Suy ra $k \in{1,2,3,4}$.

  • Nếu $k=1$ thì phương trình có dạng $x^2+y^2+x+y=x y$, tương đương với

$\quad\quad\quad\quad\quad\quad\quad\quad\left(x-\frac{y}{2}\right)^2+\frac{3}{4} y^2+x+y=0$ (vô lí vì $\left.x, y>0\right)$.

  • Nếu $k=2$ thì phương trình có dạng $x^2+y^2+x+y=2 x y$, tương đương với

$\quad\quad\quad\quad\quad\quad\quad\quad\quad\quad(x-y)^2+x+y=0$ (vô lí vì $x, y>0$ ).

  • Nếu $k=3$ thì phương trình có nghiệm $(x, y)=(2,2)$.
  • Nếu $k=4$ thì phương trình có nghiệm $(x, y)=(1,1)$.

Vậy các giá trị cần tìm là $k=3, k=4$.

Bài 8. Cho tam giác $A B C$ nội tiếp đường tròn $(O)$. Gọi $I, I_1, I_2, I_3$ lần lượt là tâm đường tròn nội tiếp và tâm đường tròn bàng tiếp các đỉnh $A, B, C$ của tam giác $A B C$. Đường tròn ngoại tiếp tam giác $I_2 I_3$ cắt $(O)$ tại hai điểm $M_1, N_1$. Gọi $J_1$ là giao điểm của $A I$ và $(O)$. Ký hiệu $d_1$ là đường thẳng qua $J_1$ và vuông góc với $M_1 N_1$. Xác định các đường thẳng $d_2, d_3$ tương tự. Chứng minh rằng $d_1, d_2, d_3$ đồng quy.

Lời giải. Gọi $\left(O^{\prime}\right)$ là đường tròn ngoại tiếp tam giác $I_1 I_2 I_3$ và $\left(O_1\right)$ là đường tròn ngoại tiếp tam giác $\left(I I_2 I_3\right)$. Ta có $A I \perp I_2 I_3, B I \perp I_3 I_1$ nên $I$ là trực tâm tam giác $I_1 I_2 I_3$ và $(O)$ là đường tròn Euler của tam giác $I_1 I_2 I_3$ nên $O$ là trung điểm của $I O^{\prime}$. Mặt khác thì

$\angle I_2 O_1 I_3=2\left(180^{\circ}-\angle I_2 I I_3\right)=2 \angle I_2 I_1 I_3=I_2 O^{\prime} I_3 .$

Do đó $O^{\prime}$ đối xứng với $O_1$ qua $I_2 I_3$, suy ra $\overrightarrow{O^{\prime} O_1}=\overrightarrow{I_1 I}$, tức $A I O_1 O^{\prime}$ là hình bình hành. Mà $O$ là trung điểm $I O^{\prime}$ nên $O$ cũng là trung điểm của $I_1 O_1$. Hơn nữa, $O O_1 \perp M_1 N_1$ (đường nối tâm vuông góc với dây cung) nên $I_1 O \perp M_1 N_1$.

Mặt khác, $J_1$ là trung điểm của $I I_1$ (đường tròn Euler đi qua trung điểm của đoạn thẳng nối trực tâm của tam giác với đỉnh của tam giác ấy) nên phép vị tự tâm $I$, tỉ số $k=\frac{1}{2}$ biến $I_1 O_1$ thành $d_1$. Do đó $d_1$ đi qua trung điểm $S$ của $O I$. Tương tự, ta suy ra $d_2, d_3$ cũng đi qua $S$, tức $d_1, d_2, d_3$ đồng quy.

Nhận xét. Bài toán thực chất là việc đổi mô hình từ một tính chất quen thuộc liên quan đến trực tâm, chân đường cao sang mô hình ba tâm bàng tiếp. Vì thế, đôi khi việc chuyển đổi giữa các mô hình giúp cho bài toán sáng sủa, dễ xử lý hơn.

 

 

 

 

 

 

 

 

 

 

 

 

 

 

 

 

 

 

 

 

 

 

 

 

 

 

 

 

 

 

 

 

 

 

 

 

 

 

 

 

 

 

 

 

 

 

 

 

 

 

 

 

 

 

 

 

 

 

 

 

 

 

 

 

 

 

 

 

 

 

 

 

 

 

 

 

 

 

 

 

 

 

Đề thi và đáp án kì thi chọn đội tuyển thi Quốc gia trường Phổ thông Năng khiếu năm học 2008 – 2009

ĐỀ THI

Ngày thi thứ nhất

Bài 1.

(a) Chứng minh rằng tồn tại số $n$ chẵn, $n>2008$ sao cho $2009 n-49$ là số chính phương.

(b) Chứng minh rằng không tồn tại số nguyên $m$ sao cho $2009 m-147$ là số chính phương.

Bài 2.

(a) Tìm số các số tự nhiên có 3 chữ số khác nhau và chia hết cho 6 .

(b) Tìm số các số tự nhiên có $n$ chữ số lập từ các chữ số ${1,2,3,4,5}$ và chia hết cho 3 .

Bài 3. Cho tam giác $A B C$ có $A$ cố định và $B, C$ thay đổi trên đường thẳng $d$ cố định sao cho nếu gọi $A^{\prime}$ là hình chiếu của $A$ lên $d$ thì $\overline{A^{\prime} B} \cdot \overline{A^{\prime} C}<0$ và không đổi. Gọi $M$ là hình chiếu của $A^{\prime}$ lên $A C$.

(a) Chứng minh rằng tâm $I$ đường tròn ngoại tiếp tam giác $B C M$ thuộc một đường thẳng cố định.

(b) Gọi $N$ là hình chiếu của $A^{\prime}$ lên $A B$ và $K$ là giao điểm hai tiếp tuyến của $\left(A^{\prime} M N\right)$ tại $M, N$. Chứng minh rằng $K$ thuộc một đường thẳng cố định.

Bài 4. Cho $f(x)=x^2+a x+b$ là tam thức bậc hai với $a, b \in \mathbb{R}$ và $f(f(x))=0$ có 4 nghiệm thực phân biệt. Biết rằng tổng của 2 nghiệm nào đó trong số 4 nghiệm đã nêu bằng $-1$. Chứng minh rằng $b \leq-\frac{1}{4}$.

Ngày thi thứ hai

Bài 5. Giả sử đa thức $P(x)=(x+1)^p(x-3)^q=x^n+a_1 x^{n-1}+a_2 x^{n-2}+\cdots+a_n$, trong đó $p, q$ là các số nguyên dương. Chứng minh rằng nếu như $a_1=a_2$ thì $3 n$ là một số chính phương.

Bài 6.

(a) Cho $a, b, c$ là các số thực dương. Chứng minh rằng ta có bất đẳng thức

$\quad\quad\quad\quad\quad\quad\quad\quad\quad\quad\quad\frac{a^2+b^2+c^2}{a b+b c+c a}+\frac{8 a b c}{(a+b)(b+c)(c+a)} \geq 2 .$

(b) Chứng minh rằng tồn tại các số thực dương $a, b, c$ sao cho

$\quad\quad\quad\quad\quad\quad\quad\quad\quad\quad\quad\frac{a b+b c+c a}{a^2+b^2+c^2}+\frac{(a+b)(b+c)(c+a)}{8 a b c}<2 .$

Bài 7. Cho góc $O x y$ và một điểm $P$ nằm bên trong nó. Gọi $\gamma$ là đường tròn thay đổi nhưng luôn qua $O$ và $P$. Giả sử $\gamma$ cắt $O x, O y$ tại $M, N$. Tìm quỹ tích trọng tâm $G$ và trực tâm $H$ của tam giác $O M N$.

Bài 8. Với mỗi số nguyên dương gọi là tổng các chữ số của $n$.

(a) Chứng minh rằng $n=999, n=2999$ không thể biểu diễn thành tổng $a+b$ mà $S(a)=S(b)$.

(b) Chứng minh rằng với mọi $n$ mà $999<n<2999$ thì điều kiện trên được thỏa mãn.

LỜI GIẢI

Ngày thi thứ nhất

Bài 1.

(a) Chứng minh rằng tồn tại số $n$ chẵn, $n>2008$ sao cho $2009 n-49$ là số chính phương.

(b) Chứng minh rằng không tồn tại số nguyên $m$ sao cho $2009 m-147$ là số chính phương.

Lời giải. Chú ý rằng $2009=49 \cdot 41=7^2 \cdot 41$ nên yêu cầu bài toán tương đương với việc chứng minh

  1. Tồn tại số $n$ chẵn, $n>2008$ sao cho $41 n-1$ là số chính phương.
  2. Không tồn tại số nguyên $m$ sao cho $41 m-3$ là số chính phương.

(a) Trước hết, ta đi tìm một số $a$ sao cho $a^2+1$ chia hết cho 41 . Điều này có thể được thực hiện bằng cách thử tuần tự. Ta dễ dàng tìm được $a=9$ thỏa mãn. Từ đây, ta thấy các số $(82 k+9)^2+1$ là số chẵn và chia hết cho 41 . Bây giờ chỉ cần chọn.

$\quad\quad\quad\quad\quad\quad\quad\quad\quad\quad\quad n=\frac{(82 k+9)^2+1}{41}$

với $k$ đủ lớn là ta tìm được số $n$ thỏa mãn điều kiện đề bài.

(b) Giả sử tồn tại $m$ sao cho $41 m-3=a^2$. Khi đó ta có $-3 \equiv a^2(\bmod 41)$. Từ đó theo định lý Fermat nhỏ thì

$\quad\quad\quad\quad\quad\quad\quad\quad\quad\quad\quad (-3)^{20} \equiv a^{40} \equiv 1 \quad(\bmod 41) .$

Nhưng mặt khác, ta lại có $(-3)^4 \equiv-1(\bmod 41)$, suy ra

$\quad\quad\quad\quad\quad\quad\quad\quad\quad\quad\quad (-3)^{20} \equiv(-1)^5 \equiv-1 \quad(\bmod 41) .$

Do đó $1 \equiv-1(\bmod 41)$, vô lý. Do đó điều đã giả sử là sai, tức là không tồn tại số nguyên $m$ sao cho $41 m-3$ là số chính phương.

Bài 2.

(a) Tìm số các số tự nhiên có 3 chữ số khác nhau và chia hết cho 6 .

(b) Tìm số các số tự nhiên có $n$ chữ số lập từ các chữ số ${1,2,3,4,5}$ và chia hết cho 3 .

Lời giải. (a) Ta sẽ tìm các số có dạng $\overline{a b c}$ sao cho $c$ chẵn và $3 \mid a+b+c$. Ta có các trường hợp sau

  • Nếu $c=0$ thì $a+b \in{3,6,9,12,15,18}$ nên có $2+4+8+6+4+0=24$.
  • Nếu $c=2$ thì $a+b \in{1,4,7,10,13,16}$ nên có $1+3+5+6+6+2=23$.
  • Nếu $c=4$ thì $a+b \in{2,5,8,11,14,17}$ nên có $1+3+7+6+4+2=23$.
  • Nếu $c=6$ thì $a+b \in{3,6,9,12,15,18}$ nên có $3+4+7+6+2+0=22$.
  • Nếu $c=8$ thì $a+b \in{1,4,7,10,13,16}$ nên có $1+3+7+6+4+2=23$.

Do đó, có tất cả $24+23+23+22+23=115$ số thỏa mãn đề bài.

(b) Ta xét khai triển tương ứng

$\quad\quad\quad\quad\quad\quad\quad\quad\quad\quad\quad P(x)=\left(x+x^2+x^3+x^4+x^5\right)^n=\sum_{k=n}^{5 n} a_k x^k$

Số các số chia hết cho ba cần tìm chính bằng tổng các số hạng của khai triển trên có dạng $x^{3 k}$, giả sử tổng đó là $A$. Xét ba nghiệm phức của phương trình $t^3=1$ là $t=1, t=\varepsilon, t=\varepsilon^2$. Ta có:

$\quad\quad\quad\quad\quad\quad\quad\quad\quad\quad\quad P(1)=5^n, P(\varepsilon)=\left(\varepsilon+\varepsilon^2+\varepsilon^3+\varepsilon+\varepsilon^2\right)^n =(-1)^n $

$\quad\quad\quad\quad\quad\quad\quad\quad\quad\quad\quad P\left(\varepsilon^2\right)=\left(\varepsilon^2+\varepsilon+1+\varepsilon^2+\varepsilon\right)^n =(-1)^n$

Hơn nữa, dễ dàng thấy rằng

$\quad\quad\quad\quad\quad\quad\quad\quad\quad\quad\quad P(1)+P(\varepsilon)+P\left(\varepsilon^2\right)=\sum_{k=n}^{5 n} a_k\left(1+\varepsilon^k+\varepsilon^{2 k}\right) $

Nếu $3 \mid k$ thì $P(1)+P(\varepsilon)+P\left(\varepsilon^2\right)=3 A$. Nếu $k$ không chia hết cho 3 thì $1+\varepsilon^k+\varepsilon^{2 k}=$ $1+\varepsilon+e^2=0$ nên các biểu thức còn lại trong tổng $P(1)+P(\varepsilon)+P\left(\varepsilon^2\right)$ đều bằng 0. Vậy nên ta tính được

$\quad\quad\quad\quad\quad\quad\quad\quad\quad\quad\quad P(1)+P(\varepsilon)+P\left(\varepsilon^2\right)=3 A \Leftrightarrow A=\frac{5^n+2(-1)^n}{3}$

Đó cũng chính là số các số cần tìm.

Bài 3. Cho tam giác $A B C$ có $A$ cố định và $B, C$ thay đổi trên đường thẳng $d$ cố định sao cho nếu gọi $A^{\prime}$ là hình chiếu của $A$ lên $d$ thì $\overline{A^{\prime} B} \cdot \overline{A^{\prime} C}<0$ và không đổi. Gọi $M$ là hình chiếu của $A^{\prime}$ lên $A C$.

(a) Chứng minh rằng tâm $I$ đường tròn ngoại tiếp tam giác $B C M$ thuộc một đường thẳng cố định.

(b) Gọi $N$ là hình chiếu của $A^{\prime}$ lên $A B$ và $K$ là giao điểm hai tiếp tuyến của $\left(A^{\prime} M N\right)$ tại $M, N$. Chứng minh rằng $K$ thuộc một đường thẳng cố định.

Lời giải. (a) Đặt $\overline{A^{\prime} B} \cdot \overline{A^{\prime} C}=-k^2$. Từ $E$ hạ $I E \perp A A^{\prime}$. Gọi $N^{\prime}, P$ lần lượt là giao điểm của $(B M N)$ với $A A^{\prime}$. Ta có:

$\quad\quad\quad\quad\quad\quad\quad\quad\overline{A M} \cdot \overline{A B}=\overline{A N^{\prime}} \cdot \overline{A P}=A A^{\prime 2}=\left(\overline{A A^{\prime}}+\overline{A^{\prime} N^{\prime}}\right)\left(\overline{A A^{\prime}}+\overline{A^{\prime} P}\right)$

Do đó,

$\quad\quad\quad\quad\quad\quad\quad\quad\quad\quad\quad \overline{A A^{\prime}}\left(\overline{A^{\prime} N^{\prime}}+\overline{A^{\prime} P}\right)=-\overline{A^{\prime} N^{\prime}} \cdot \overline{A^{\prime} P}=k^2$

Ta thu dược

$\quad\quad\quad\quad\quad\quad\quad\quad\quad\quad\quad\quad\quad\quad\quad 2 \overline{A^{\prime} E}=\frac{k^2}{\overline{A A^{\prime}}}$

Suy ra $E$ cố định. Vậy điểm $I$ chạy trên đường thẳng qua $E$ và vuông góc với $A A^{\prime}$ cố định.

(b) Gọi $F$ là trung điểm của $A A^{\prime}$, ta có $F$ là tâm đường tròn ngoại tiếp tứ giác $A N A^{\prime} M$. Gọi $Z$ là giao điểm của $M N$ và $A A^{\prime}$, ta có

$\quad\quad\quad\quad\quad\quad\quad\quad\quad\quad\quad \overline{Z A} \cdot \overline{Z A^{\prime}}=\overline{Z M} \cdot \overline{Z N^{\prime}}=-k^2 \text {. }$

Suy ra $Z$ cố định.

Bây giờ, từ $K$ hạ $K Y$ vuông góc với $A A^{\prime}$. Ta có $F, M, N, K, Y$ cùng nằm trên một đường tròn, suy ra

$\quad\quad\quad\quad\quad\quad\quad\quad\quad\quad\quad \overline{Z F} \cdot \overline{Z Y}=\overline{Z M} \cdot \overline{Z N^{\prime}}=-k^2 .$

Từ đây dễ thấy $Y$ cố định. Vậy $K$ di động trên đường thẳng qua $Y$ vuông góc với $A A^{\prime}$ cố định.

Bài 4. Cho $f(x)=x^2+a x+b$ là tam thức bậc hai với $a, b \in \mathbb{R}$ và $f(f(x))=0$ có 4 nghiệm thực phân biệt. Biết rằng tổng của 2 nghiệm nào đó trong số 4 nghiệm đã nêu bằng $-1$. Chứng minh rằng $b \leq-\frac{1}{4}$.

Lời giải. Trước hết, dễ thấy $f(x)=0$ phải có 2 nghiệm phân biệt, đặt là $c_1<c_2$. Gọi $x_1, x_2$ là 2 trong số 4 nghiệm có tổng bằng $-1$. Theo định lý Viete thì: $c_1+c_2=$ $-a, c_1 c_2=b$. Ngoài ra,

$\quad\quad\quad\quad\quad\quad\quad\quad\quad\quad\quad f(f(x))=0 \Leftrightarrow f(x)=c_1 \vee f(x)=c_2 .$

Ta xét các trường hợp sau

  • Nếu $x_1, x_2$ là nghiệm của cùng một phương trình trong hai phương trình trên, theo định lý Viete thì $-a=x_1+x_2=-1$ nên $a=1$. Do $f(x)=c_1, f(x)=c_2$ đều phải có 2 nghiệm phân biệt nên $\Delta_1>0, \Delta_2>0$, tức là $1-4\left(b-c_1\right)>$ $0,1-4\left(b-c_2\right)>0$. Cộng lại, ta có

$\quad\quad\quad\quad\quad\quad\quad\quad\quad\quad\quad 2-4\left(2 b-c_1-c_2\right)>0 \Leftrightarrow 1-2(2 b+1)>0 \Leftrightarrow b<-\frac{1}{4} .$

  • Nếu $x_1, x_2$ là nghiệm của hai phương trình thì $x_1^2+a x_1+b=c_1, x_2^2+a x_2+b=c_2$. Cộng lại, ta có $x_1^2+x_2^2+a\left(x_1+x_2\right)+2 b=c_1+c_2 \Leftrightarrow x_1^2+x_2^2+2 b=0$. Do đó $b=-\frac{x_1^2+x_2^2}{2} \leq-\frac{\left(x_1+x_2\right)^2}{4}=-\frac{1}{4}$.

Trong mọi trường hợp, ta luôn có điều phải chứng minh.

Ngày thi thứ hai

Bài 5. Giả sử $P(x)=(x+1)^p(x-3)^q=x^n+a_1 x^{n-1}+a_2 x^{n-2}+\cdots+a_n$, trong đó $p, q$ là các số nguyên dương. Chứng minh rằng nếu $a_1=a_2$ thì $3 n$ là một số chính phương.

Lời giải. Ta có

$P(x)=(x+1)^p(x-3)^q=\left(x^p+C_p^1 x^{p-1}+C_p^2 x^{p-2}+\cdots\right)\left(x^q-3 C_q^1 x^{q-1}+9 C_q^2 x^{q-2}+\cdots\right)$

Từ đó suy ra

$\quad\quad\quad\quad\quad\quad\quad\quad\quad\quad\quad a_1=C_p^1-3 C_q^1 \text { và } a_2=C_p^2+9 C_q^2-3 C_p^1 C_q^1$

Như vậy $a_1=a_2$ khi và chỉ khi

$\quad\quad\quad\quad\quad\quad\quad\quad\quad\quad\quad p-3 q=\frac{p(p-1)}{2}+\frac{9 q(q-1)}{2}-3 p q$

hay

$\quad\quad\quad\quad\quad\quad 2 p-6 q=p^2-p+9 q^2-9 q-6 p q \text { tức là } 3 n=3(p+q)=(p-3 q)^2 \text {. }$

Suy ra $3 n$ là số chính phương. Ta có điều phải chứng minh.

Bài 6.

(a) Cho $a, b, c$ là các số thực dương. Chứng minh rằng ta có bất đẳng thức

$\quad\quad\quad\quad\quad\quad\quad\quad\quad\quad\quad \frac{a^2+b^2+c^2}{a b+b c+c a}+\frac{8 a b c}{(a+b)(b+c)(c+a)} \geq 2 .$

(b) Chứng minh rằng tồn tại các số thực dương $a, b, c$ sao cho

$\quad\quad\quad\quad\quad\quad\quad\quad\quad\quad\quad \frac{a b+b c+c a}{a^2+b^2+c^2}+\frac{(a+b)(b+c)(c+a)}{8 a b c}<2 .$

Lời giải. (a) Không mất tính tổng quát, ta có thể giả sử $c=\min {a, b, c}$. Khi đó, với chú ý rằng $a^2+b^2+c^2 \geq a b+b c+c a$, ta có

$\quad\quad\quad\quad\quad\quad\quad\quad\quad\quad \frac{a^2+b^2+c^2}{a b+b c+c a} =1+\frac{a^2+b^2+c^2-a b-b c-c a}{a b+b c+c a} $

$\quad\quad\quad\quad\quad\quad\quad\quad\quad\quad\quad\quad\quad\quad \geq 1+\frac{a^2+b^2+c^2-a b-b c-c a}{a b+b c+c a+c^2} $

$\quad\quad\quad\quad\quad\quad\quad\quad\quad\quad\quad\quad\quad\quad =\frac{a^2+b^2+2 c^2}{a b+b c+c a+c^2} $

$\quad\quad\quad\quad\quad\quad\quad\quad\quad\quad\quad\quad\quad\quad =\frac{a^2+b^2+2 c^2}{(a+c)(b+c)}$

Do đó, ta chỉ cần chứng minh.

$\quad\quad\quad \frac{a^2+b^2+2 c^2}{(a+c)(b+c)}+\frac{8 a b c}{(a+b)(b+c)(c+a)} \geq 2$

Bất đẳng thức này tương đương với

$a^3+b^3+a^2 b+b^2 a+2 c^2 a+2 c^2 b+8 a b c \geq 2\left(a^2 b+a^2 c+b^2 a+b^2 c+c^2 a+c^2 b+2 a b c\right)$

hay

$a^3+b^3+4 a b c \geq a^2 b+b^2 a+2 a^2 c+2 b^2 c . \Leftrightarrow(a-b)^2(a+b-2 c) \geq 0 .$

Do $c=min (a, b, c)$ nên bất đẳng thức cuối hiển nhiên đúng.

Phép chứng minh hoàn tất. Đẳng thức xảy ra khi và chỉ khi $a=b=c$.

(b) Kiểm tra trực tiếp, ta thấy bộ $(a, b, c)=(2,1,1)$ thỏa mãn đề bài. Điều này cho thấy rằng nếu nghịch đảo cả hai phân số trong vế trái của câu a) thì bài toán không còn đúng nữa.

Bài 7. Cho góc $O x y$ và một điểm $P$ nằm bên trong nó. Gọi $\gamma$ là đường tròn thay đổi nhưng luôn qua $O$ và $P$. Giả sử $\gamma$ cắt $O x, O y$ tại $M, N$. Tìm quỹ tích trọng tâm $G$ và trực tâm $H$ của tam giác $O M N$.

Lời giải. (a) Quĩ tích trọng tâm G của tam giác $O M N$

Gọi $I$ là trung điểm $M N$. Ta có ${G}=V_O^{\frac{2}{3}}({I})$. Ta sẽ tìm quỹ tích điểm $I$.

Phần thuận. Gọi $X$ là giao điểm thứ hai của $(I M P)$ với $O x, Y$ là giao điểm thứ hai của $(I N P)$ với $O y$. Ta có:

$\quad\quad\quad\quad\quad\quad\quad\quad\quad\quad\quad \angle X I P=\angle X M P=\angle P N Y=180^{\circ}-\angle P I Y$

do đó $X, Y, Z$ thẳng hàng. Mặt khác,

$\quad\quad\quad\quad\quad\quad\quad\quad\quad\quad\quad \angle I X P=\angle N M P=\angle P O N$

nên $\angle I X P$ không đổi. Tương tự ta cũng có $\angle X Y P$ không đổi mà $P$ cố định suy ra $X, Y$ cố định vậy $I$ nằm trên đường thẳng $X Y$ cố định.

Phần đảo. Lấy $X, Y$ lần lượt thuộc tia $O x, O y$ sao cho $\angle P X Y=\angle P O y$ và $\angle P Y X=$ $\angle P O x$. Lấy $I \in X Y$ ta sẽ chứng minh tồn tại $M \in O x, N \in O y$ sao cho $(O M N)$ đi qua $P$. Thật vậy,

Gọi $M$ là giao điểm thứ hai của $(I X P)$ và $O x, N$ là giao điểm thứ hai của $(I Y P)$ và $O y$. Ta có:

$\quad\quad\quad\quad\quad\quad\quad\quad\quad\quad\quad \angle X M P=\angle X I P=\angle P N Y$

nên tứ giác $O M P N$ nội tiếp. Ta có điều phải chứng minh.

(b) Quỹ tích trục tâm H của tam giác OMN.

Phần thuận. Gọi $T$ là trung điểm $O P . X, Y$ là hình chiếu của $P$ lên $O x, O y . K$ là trực tâm tam giác $O X Y . I$ là tâm đường tròn $(O M N)$. Ta có

$\quad\quad\quad\quad\quad\quad\quad\quad\quad\frac{O K}{O T}=\frac{O H}{O I}(=2|\cos \angle x O y|) \text { và } \angle I O T=\angle H O K$

nên $\triangle I O T \sim \triangle H O K$. Mà $\angle I T O=90^{\circ}$ nên $\angle H K O=90^{\circ}$. Vậy $H$ thuộc đường thẳng qua $K$ vuông góc với $O K$ (Chú ý $K$ cố định).

Phần đảo. Lấy $H$ thuộc đường thẳng qua $K$ vuông góc với $O K$, trong đó $K$ là trực tâm tam giác $O X Y$ và $X, Y$ là hình chiếu của $P$ lên $O x, O y$. Ta sẽ chứng minh tồn tại $M \in O x, N \in O y$ sao cho $(O M N)$ đi qua $P$ và $\triangle O M N$ nhận $H$ là trực tâm. Thật vậy, gọi $T$ là trung điểm $O P$ và dựng $\triangle O T I \sim \triangle O K H$. Ta có

$\quad\quad\quad\quad\quad\quad\quad\quad\quad\quad\quad \angle O T I=\angle O K H=90^{\circ}$

$\quad\quad\quad\quad\quad\quad\quad\quad\quad\quad\quad \angle I O T=\angle H O K$

nên $I$ nằm trên trung trực của $O P$. Do đó nếu vẽ $(T, T O)$ thì đường tròn này đi qua $P$ và cắt $O x, O y$ tại $M, N$. Ta có $K, T$ là trực tâm và tâm đường tròn $(O X Y)$ nên $\angle K O y=\angle T O x$. Ta được

$\quad\quad\quad\quad\quad\quad\quad\quad\quad\quad\quad \angle I O y=\angle H O x$

Lại có $\triangle O T I \sim \triangle O K H$ nên $\frac{O H}{O I}=\frac{O T}{O K}=2|\cos \angle x O y|$. Ta suy ra $H$ là trực tâm tam giác $O M N$

Bài 8. Với mỗi số nguyên dương gọi là tổng các chữ số của $n$.

(a) Chứng minh rằng $n=999, n=2999$ không thể biểu diễn thành tổng $a+b$ mà $S(a)=S(b)$.

(b) Chứng minh rằng với mọi $n$ mà $999<n<2999$ thì điều kiện trên được thỏa mãn.

Lời giải. Ta sẽ giải bài toán tồng quát sau: Tồn tại các số $a, b$ thỏa mãn điều kiện khi và chỉ khi số $n>1$ không có dạng $n=\overline{m 999 \ldots 9}$ với $0 \leq m \leq 8$ và $S(n)$ lẻ.

Thật vậy, xét $n=\overline{d_1 d_2 d_3 \ldots d_k}$ với $k$ là số các chữ số của $n$.

Ta có 2 trường hợp cần xét như sau:

  1. Nếu $S(n)$ chẵn (tương đương với có chẵn chữ số lẻ trong trong $n$ ), ta thực hiện như sau:
  • Nếu $d_i$ chẵn thì tách thành $d_i=\frac{d_i}{2}+\frac{d_i}{2}$ và chữ số ở hàng tương ứng của $a, b$ sẽ là 2 số này.
  • Nếu $d_i$ lẻ thì tách thành $d_i=\frac{d_i-1}{2}+\frac{d_i+1}{2}$ thì $\frac{d_i+1}{2}-\frac{d_i-1}{2}=1$ và như thế, ta luân phiên thay đổi các số lớn nhỏ để ghép vào $a, b$ để đảm bảo có $S(a)=S(b)$.

Do có chẵn số $d_i$ lẻ như thế nên quá trình trên thực hiện được và trong trường hợp này, tồn tại số $a, b$ thỏa mãn.

  1. Nếu $S(n)$ lẻ (tương đương với có lẻ chữ số lẻ trong trong $n$ ). Nếu $n$ có dạng $m 999 \ldots 9$ với $0 \leq m \leq 8$ thì rõ ràng khi tách ra thành 2 phần, các phép tính tồng phía sau để thu được các sồ 9 là không có nhớ và chúng có dạng $d_i+d_i^{\prime}=9$. Khi đó, ta có

$\quad\quad\quad\quad\quad\quad\quad S(a)+S(b)=S(n) \text { mà }\left\{\begin{array}{l}S(a)=S(b) \\ S(n) \equiv 1(\bmod 2)\end{array}\right.$

Điều mâu thuân trên cho thấy trường hợp này không tồn tại cách tách $n$ thành $a, b$ thỏa mãn. Nếu $n$ không có dạng trên thì dễ thấy tồn tại $d_i \neq 0, d_{i+1} \neq 9$.

Ta viết lại $n$ như sau:

$n=\overline{d_1 d_2 d_3 \ldots d_i d_{i+1} \ldots d_k}=\overline{d_1 d_2 d_{3 \ldots}\left(d_i-1\right) 9 \ldots d_k}+\left(d_{i+1}+1\right) 10^{k-d_i}$

Đặt $a=\overline{d_1 d_2 d_3 \ldots\left(d_i-1\right) 9 \ldots d_k}, b=\left(d_{i+1}+1\right) 10^{k-d_i}$ thì $S(a), S(b)$ cùng tính chẵn lẻ và $a+b=n$. Ta có thể giả sử $S(a)>S(b)$, trường hợp còn lại chứng minh tương tự. Nếu chọn một vị trí $t$ mà $d_t \neq 0$ thì có thể đổi các số $a, b$ thành

$a^{\prime}=\overline{d_1 d_2 d_3 \ldots\left(d_i-1\right) 9 \ldots\left(d_t-1\right) \ldots d_k}, y=\left(d_{i+1}+1\right) 10^{k-i}+10^{k-t}$

với $k \neq i$. Khi đó

$\quad\quad\quad \quad\quad\quad S\left(a^{\prime}\right)-S\left(b^{\prime}\right)=S(a)-1-(S(b)+1)=-2 .$

Cứ như vậy, ta thực hiện liên tiếp đến khi nào chênh lệch giữa hai tổng các chữ số bằng 0 thì dừng lại (tồn tại thời điểm như vậy vì ban đầu chúng cùng tính chẵn lẻ nên hiệu của chúng là số chẵn và mỗi lần thực hiện quá trình trên thì hiệu giảm đi 2 đơn vị). Các số $a, b$ lúc đó sẽ thỏa mãn đề bài và cũng tồn tại cách tách.

Bài toán được giải quyết hoàn toàn.

 

 

 

 

 

 

 

 

 

 

 

 

 

 

 

 

 

 

 

 

 

 

 

 

 

 

 

 

 

 

 

 

 

 

ĐỀ THI VÀO CHUYÊN TOÁN LỚP 10 TP.HCM 2012

Bài 1. Giải phương trình:

$\sqrt{8 x+1}+\sqrt{46-10 x}=-x^{3}+5 x^{2}+4 x+1$

Bài 2. Cho đa thức $f(x)=a x^{3}+b x^{2}+c x+d$ với $a$ là số nguyên dương, biết $f(5)-$ $f(4)=$ 2012. Chứng minh rằng: $f(7)-f(2)$ là hợp số.

Bài 3. Cho ba số dương $a, b, c$ thỏa $a+b+c=1$. Tìm giá trị nhỏ nhất của biểu thức:

$A=14\left(a^{2}+b^{2}+c^{2}\right)+\frac{a b+b c+c a}{a^{2} b+b^{2} c+c^{2} a}$

Bài 4. Cho tứ giác $A B C D$ nội tiếp đường tròn $(O, R)$ có $A C$ vuông góc với $B D$ tại $H$. Trên cạnh $A B$ lấy điểm $M$ sao cho $A B=3 A M$. Trên cạnh $H C$ lấy trung điểm $N$. Chứng minh rằng $M H$ vuông góc với $D N$.

Bài 5. Cho đường tròn tâm $O$ và đường tròn tâm $I$ cắt nhau tại hai điểm $A$ và $B(O$ và $I$ nằm khác phía đối với đường thẳng $A B), I B$ cắt $(O)$ tại $E, O B$ cắt $(I)$ tại $F$. Qua $B$ vẽ đường thẳng $M N$ song song với $E F(M$ thuộc $(O), N$ thuộc $(I)$ ).

(a) Chứng minh rằng $O A I E$ nội tiếp.

(b) Chứng minh rằng: $A E+A F=M N$.

Bài 6. Trên mặt phẳng cho 2013 điểm tùy ý sao cho trong ba điểm bất kì thì tồn tại 2 điểm mà khoảng cách giữa hai điểm đó luôn bé hơn 1. Chứng minh rằng tồn tại một đường tròn có bán kính bằng 1 chứa ít nhất 1007 điểm (kế cả biên).

 

LỜI GIẢI

Bài 1. Giải phương trình:

$\sqrt{8 x+1}+\sqrt{46-10 x}=-x^{3}+5 x^{2}+4 x+1$

Lời giải. $\sqrt{8 x+1}+\sqrt{46-10 x}=-x^{3}+5 x^{2}+4 x+1$

ĐKХĐ: $\frac{-1}{8} \leq x \leq \frac{23}{5}$

Sử dụng lượng liên hợp, phương trình ban đầu tương đương với:

$\sqrt{8 x+1}-3+\sqrt{46-10 x}-6+x^{3}-x^{2}-4 x^{2}+4 x-8 x+8=0$

$\Leftrightarrow(x-1)\left(\frac{8}{\sqrt{8 x+1}+3}-\frac{10}{\sqrt{46-10 x}+6}+x^{2}-4 x-8\right)=0$

Từ đó ta có phương trình có một nghiệm là $x=1$. Xét biểu thức:

$\frac{8}{\sqrt{8 x+1}+3}-\frac{10}{\sqrt{46-10 x}+6}+x^{2}-4 x-8=0$

Từ điều kiện ta có:

$-1<x<5 \Leftrightarrow(x+1)(x-5)<0 \Leftrightarrow x^{2}-4 x-5<0$

Lại có: $\frac{8}{\sqrt{8 x+1}+3} \leq \frac{8}{3}<\frac{9}{3}=3 \Leftrightarrow \frac{8}{\sqrt{8 x+1}+3}-3<0$ Từ đó ta có:

$\frac{8}{\sqrt{8 x+1}+3}-\frac{10}{\sqrt{46-10 x}+6}+x^{2}-4 x-8<0$

Vậy phương trình đã cho có nghiệm duy nhất là: $x=1$

Bài 2. Cho đa thức $f(x)=a x^{3}+b x^{2}+c x+d$ với $a$ là số nguyên dương, biết $f(5)-$ $f(4)=2012$. Chứng minh rằng: $f(7)-f(2)$ là hợp số.

Lời giải. Ta có: $f(x)=a x^{3}+b x^{2}+c x+d$

Từ đó ta tính được: $f(5)=125 a+25 b+5 c+d, f(4)=64 a+16 b+4 c+d$

Vậy: $f(5)-f(4)=61 a+9 b+c=2012, f(7)=343 a+49 b+7 c+d, f(2)=8 a+4 b+$ $2 c+d$

Vậy: $f(7)-f(2)=335 a+45 b+5 c=5(67 a+9 b+c)=30 a+5(61 a+9 b+c)=30 a+$ 10060

Từ đó ta có: $f(7)-f(2)$ là hợp số vì $a$ là số nguyên dương và nó chia hết cho $2,5,10$.

Bài 3. Cho ba số dương $a, b, c$ thỏa $a+b+c=1$. Tìm giá trị nhỏ nhât của biểu thức:

$A=14\left(a^{2}+b^{2}+c^{2}\right)+\frac{a b+b c+c a}{a^{2} b+b^{2} c+c^{2} a}$

Lời giải.

Cách 1:

$\left(a^{2}+b^{2}+c^{2}\right)(a+b+c)=a^{3}+b^{3}+c^{3}+\left(a^{2} b+b^{2} c+c^{2} a\right)+\left(b^{2} a+a^{2} c+c^{2} b\right) $

$\left(a^{2}+b^{2}+c^{2}\right)(a+b+c)=\left(a^{3}+a b^{2}\right)+\left(b^{3}+b c^{2}\right)+\left(c^{3}+c a^{2}\right)+\left(a^{2} b+b^{2} c+c^{2} a\right)$

Áp dụng bất đẳng thức Cauchy và do $a+b+c=1$, ta có:

$\left(a^{2}+b^{2}+c^{2}\right) \geq 2 a^{2} b+2 b^{2} c+2 c^{2} a+\left(a^{2} b+b^{2} c+c^{2} a\right)=3\left(a^{2} b+b^{2} c+c^{2} a\right)$

Mặt khác: $a b+b c+c a=\frac{1-\left(a^{2}+b^{2}+c^{2}\right)}{2}$

Từ đó ta có: $F \geq 14\left(a^{2}+b^{2}+c^{2}\right)+\frac{3-3\left(a^{2}+b^{2}+c^{2}\right)}{2\left(a^{2}+b^{2}+c^{2}\right)}$

Hay: $F \geq 14\left(a^{2}+b^{2}+c^{2}\right)+\frac{3}{2\left(a^{2}+b^{2}+c^{2}\right)}-\frac{3}{2}$

Áp dụng bất đẳng thức Cauchy, ta có:

$27\left(a^{2}+b^{2}+c^{2}\right)+\frac{3}{\left(a^{2}+b^{2}+c^{2}\right)} \geq 2 \sqrt{27\left(a^{2}+b^{2}+c^{2}\right) \cdot \frac{3}{\left(a^{2}+b^{2}+c^{2}\right)}}=18 $

$a^{2}+b^{2}+c^{2} \geq \frac{1}{3}(a+b+c)^{2}=\frac{1}{3}$

Vậy: $28\left(a^{2}+b^{2}+c^{2}\right)+\frac{3}{\left(a^{2}+b^{2}+c^{2}\right)} \geq 18+\frac{1}{3}=\frac{55}{3}$

Từ đó ta có: $F \geq \frac{55}{6}-\frac{3}{2}=\frac{23}{3}$

Đẳng thức xảy ra khi: $a=b=c=\frac{1}{3}$

Cách 2:

Do $a, b, c$ dương và $a+b+c=1$ nên ta có:

$(1-c)^{2}=(a+b)^{2} \geq 4 a b \Leftrightarrow 1-2 c+c^{2} \geq 4 a b \Leftrightarrow a-2 a c+a c^{2} \geq 4 a^{2} b $

$(1-a)^{2}=(b+c)^{2} \geq 4 b c \Leftrightarrow 1-2 a+a^{2} \geq 4 b c \Leftrightarrow b-2 a b+a^{2} b \geq 4 b^{2} c $

$(1-b)^{2}=(c+a)^{2} \geq 4 c a \Leftrightarrow 1-2 b+b^{2} \geq 4 c a \Leftrightarrow c-2 b c+b^{2} c \geq 4 a c^{2}$

Hay: $a+b+c-2(a b+b c+c a) \geq 3\left(a^{2} b+b^{2} c+c^{2} a\right)$

$\Leftrightarrow 1-2(a b+b c+c a) \geq 3\left(a^{2} b+b^{2} c+c^{2} a\right)$

Vậy: $F \geq 14[1-2(a b+b c+c a)]+\frac{3(a b+b c+c a)}{1-2(a b+b c+c a)}$

Đạt: $t=1-2(a b+b c+c a), t \geq \frac{1}{3}$

Áp dụng bất đẳng thức Cauchy ta có:

$F \geq 14 t+\frac{\frac{3}{2}(1-t)}{t}=14 t+\frac{3}{2 t}-\frac{3}{2}=\frac{1}{2} t+\frac{27}{2} t+\frac{3}{2 t}-\frac{3}{2} \geq \frac{1}{2} t+2 \sqrt{\frac{27}{2} t \cdot \frac{3}{2 t}}-\frac{3}{2}$

Vậy: $F \geq \frac{1}{2} \cdot \frac{1}{3}+9-\frac{3}{2}=\frac{23}{3}$

Đẳng thức xảy ra khi: $a=b=c=\frac{1}{3}$

Bài 4. Cho tứ giác $A B C D$ nội tiếp đường tròn $(O, R)$ có $A C$ vuông góc với $B D$ tại $H$. Trên cạnh $A B$ lấy điểm $M$ sao cho $A B=3 A M$. Trên cạnh $H C$ lấy trung điểm $N$. Chứng minh rằng $M H$ vuông góc với $D N$.

Lời giải.

  • Gọi $K, L$ lần lượt là trung điểm $B M$ và $H B, P$ là giao điểm của $H M$ và $A K$.

  • Ta có $K L$ là đường trung bình của tam giác $H M B$ nên $K L$ song song $H M$. Khi đó xét tam giác $A K L$ thì $P H$ là đường trung bình nên $P$ là trung điểm của $A K$.

  • Ta có từ $A B C D$ nội tiếp suy ra $H D \cdot H B=H A \cdot A C \Rightarrow H K \cdot H D=H A \cdot H N$, do đó $A D N K$ nội tiếp.

  • Suy ra $\angle N H Q=\angle A H P=\angle H A P=\angle H D N$, suy ra $\angle H Q N=90^{\circ}$.

Bài 5. Cho đường tròn tâm $O$ và đường tròn tâm $I$ cắt nhau tại hai điểm $A$ và $B(O$ và $I$ nằm khác phía đối với đường thẳng $A B), I B$ cắt $(O)$ tại $E, O B$ cắt $(I)$ tại $F$. Qua $B$ vẽ đường thẳng $M N$ song song với $E F(M$ thuộc $(O), N$ thuộc $(I))$.

a) Chứng minh rằng $O A I E$ nội tiếp.

b) Chứng minh rằng: $A E+A F=M N$.

Lời giải.

a) Chứng minh rằng tứ giác $A O E F$ nội tiếp

Do hai đường tròn $(\mathrm{O})$ và $(\mathrm{I})$ cắt nhau tại $A$ và $B$ nên ta có: $A$ đối xứng với $B$ qua $O I$. Vậy: $\angle O A I=\angle O B I$

Ta có tam giác $\triangle O B E$ cân tại $O$ nên $\angle O B E=\angle O E B$, do $\angle O B E+\angle O B I=180^{\circ}$ nên $\angle O E B+\angle O B I=180^{\circ}$. Từ đó ta có: $\angle O E B+\angle O A I=180^{\circ}$

Vậy tứ giác $O A I E$ là tứ giác nội tiếp. Chứng minh tương tự ta có: tứ giác $O A I F$ là tứ giác nội tiếp.

$\angle O E A=\angle O I A$ (tứ giác $O A I E$ là tứ giác nội tiếp)

$\angle O I A=\angle O F A$ (tứ giác $O A I F$ là tứ giác nội tiếp)

Vậy: $\angle O E A=\angle O F A$ nên tứ giác $O A F E$ là tứ giác nội tiếp

b) Chứng minh rằng: $M N=A E+A F$

Bài toán cần chứng minh tương đương với: $A F=B N$ và $A E=B M$.

Ta chỉ cần chứng minh $A F=B N$ vì $A E=B M$ là điều tương tự.

Để chứng minh $A F=B N$. Ta chỉ cần chứng minh số đo cung $\mathrm{AF}$ bằng số đo cung $\mathrm{BN}(A F, B N$ lần lượt là dây căng cung $\mathrm{AF}$, cung $\mathrm{BN}$ trong đường tròn (I)). Hay chỉ cần chứng minh: số đo cung $\mathrm{AB}$ bằng số đo cung FN. Từ đó ta chứng minh: $\angle O F A=\angle F B N$ là bài toán được giải quyết.

Do $E F | M N$ nên ta có: $\angle O F E=\angle F B N$

Mà $\angle O F E=\angle O A E=\angle O E A=\angle O F A$ (tứ giác $A O E F$ là tứ giác nội tiếp)

Từ đó ta có: $\angle O F A=\angle F B N$ (đpcm)

Bài 6. Trên mặt phẳng cho 2013 điểm tùy ý sao cho trong ba điểm bất kì thì tồn tại 2 điểm mà khoảng cách giữa hai điểm đó luôn bé hơn 1. Chứng minh rằng tồn tại một đường tròn có bán kính bằng 1 chứa ít nhất 1007 điểm (kế cả biên).

Lời giải. Gọi $A$ là một điểm bất kì trong 2013 điểm trên. Lấy $A$ làm tâm vẽ đường tròn có bán kính bằng 1 .

Nếu 2012 điểm còn lại thuộc đường tròn $(A)$ thì bài toán được chứng minh xong. Giả tồn tại một số điểm nằm ngoài đường tròn tâm $(A)$. Lấy điểm $(B)$ bất kì trong các điểm đó và vẽ đường tròn tâm $(B)$ có bán kính bằng 1 .

Giả sử tồn tại một điểm $C$ nằm ngoài hai đường tròn $(A)$ và $(B)$ thì $A B, A C$ đều lớn hơn 1. Điều này vô lí.

Từ đó ta có tất cả các điểm đã cho đều thuộc trong hai đường tròn $(A)$ và $(B)$.

Theo nguyên lí Dirichlet sẽ tồn tại một đường tròn chứa $\frac{2012}{2}+1=1007$ điểm (đpcm).

 

 

 

 

 

 

 

 

 

 

 

 

 

 

 

 

 

 

 

 

 

 

 

 

 

 

 

 

 

 

 

 

 

 

 

 

 

 

 

 

 

 

 

 

 

 

 

 

 

 

 

ĐỀ THI VÀO LỚP 10 CHUYÊN TOÁN TP.HCM 2013

Bài 1. (a) Giải phương trình: $x \sqrt{2 x-2}+5 x=9$.

(b) Cho $x, y, z$ đôi một khác nhau thỏa mãn: $\frac{1}{x}+\frac{1}{y}+\frac{1}{z}=0$. Tính giá trị biểu thực:

$P=\frac{y z}{x^{2}+2 y z}+\frac{z x}{y^{2}+2 z x}+\frac{x y}{z^{2}+2 x y}$

Bài 2. Cho phương trình $x^{2}-5 m x-4 m=0$.

(a) Định $m$ để phương trình có hai nghiệm phân biệt.

(b) Gọi $x_{1}, x_{2}$ là hai nghiệm của phương trình. Tìm $m$ để biểu thức sau đạt giá trị nhỏ nhất:

$\frac{m^{2}}{x_{1}^{2}+5 m x_{2}+12 m}+\frac{x_{2}^{2}+5 m x_{1}+12 m}{m^{2}}$

Bài 3. Cho tam giác $\triangle A B C$ có $B C$ là cạnh dài nhất. Trên $B C$ lấy hai điểm $D$ và $E$ sao cho $B D=B A, C E=C A$. Đường thẳng qua $D$ song song với $A B$ cắt $A C$ tại $M$. Đường thẳng qua $E$ song song với $A C$ cắt $A B$ tại $N$. Chứng minh rằng $A M=A N$.

Bài 4. Cho $x, y$ là hai số dương thỏa mãn: $x+y=1$. Chứng minh: $3(3 x-2)^{2}+\frac{8 x}{y} \geq$ $7 .$

Bài 5. Từ một điểm $A$ bên ngoài đường tròn $(O)$ vẽ các tiếp tuyến $A B, A C$ và cát tuyến $A E F$ (EF không đi qua $O, B$ và $C$ là các tiếp điểm). Gọi $D$ là điểm đôi xứng của $B$ qua $O . D E, D F$ lần lượt cắt $A O$ tại $M$ và $N$. Chứng minh rằng :

(a) Hai tam giác $\triangle C E F$ và $\triangle C M N$ đồng dạng.

(b) $O M=O N$.

Bài 6. Chữ số hàng đơn vị trong hệ thập phân của số $M=a^{2}+a b+b^{2}$ là $0\left(a ; b \in N^{*}\right)$.

(a) Chứng minh rằng $M$ chia hết cho 20 .

(b) Tìm chữ số hàng chục của $M$.

LỜI GIẢI

Bài 1.

a) Giải phương trình: $x \sqrt{2 x-2}+5 x=9$.

b) Cho $x, y, z$ đôi một khác nhau thỏa mãn: $\frac{1}{x}+\frac{1}{y}+\frac{1}{z}=0$. Tính giá trị biểu thực:

$P=\frac{y z}{x^{2}+2 y z}+\frac{z x}{y^{2}+2 z x}+\frac{x y}{z^{2}+2 x y}$

Lời giải.

a) Giải phương trình: $x \sqrt{2 x-2}+5 x=9$

ĐKXĐ: $x \geq 1$. Đặt $a=\sqrt{2 x-2}$ (ĐKXĐ: $a \geq 0$ )

Phương trình đã cho tương đương với:

$a x=9-5 x=9-\frac{5}{2}\left(a^{2}+2\right)=4-\frac{5}{2} a^{2}$

Ta có hệ phương trình sau:

$\left\{\begin{array} { l }{ 5 a ^ { 2 } + 2 a x = 8 } \\{ a ^ { 2 } – 2 x = – 2 }
\end{array} \Leftrightarrow \left\{\begin{array}{l}
x=\frac{a^{2}+2}{2} \\x=\frac{9}{a+5}
\end{array}\right.\right.$

$\Leftrightarrow \frac{9}{a+5}=\frac{a^{2}+2}{2} \Leftrightarrow a^{3}+5 a^{2}+2 a-8=0 \Leftrightarrow(a-1)(a+2)(a+4)=0$

Kết hợp với: ĐKXĐ: $a \geq 0$. Từ đó ta tính được: $a=1 \Leftrightarrow x=\frac{3}{2}$

b) Tính giá trị biểu thức: $P=\frac{y z}{x^{2}+2 y z}+\frac{z x}{y^{2}+2 z x}+\frac{x y}{z^{2}+2 x y}$

Từ điều kiện của đề bài ta có: $x y+y z+z x=0$

Thêm vào đó: $x^{2}+2 y z=x^{2}+y z-x y-x z=(x-y)(x-z)$

Từ đó ta có:

$P=\sum_{x, y, z} \frac{y z}{x^{2}+2 y z}=\sum_{x, y, z} \frac{y z}{(x-y)(x-z)}=-\frac{y z(y-z)+x z(z-x)+x y(x-y)}{(x-y)(y-z)(z-x)}$

Vậy: $P=1$

Bài 2. Cho phương trình $x^{2}-5 m x-4 m=0$.

a) Định $m$ để phương trình có hai nghiệm phân biệt.

b) Gọi $x_{1}, x_{2}$ là hai nghiệm của phương trình. Tìm $m$ để biểu thức sau đạt giá trị nhỏ nhất:

$\frac{m^{2}}{x_{1}^{2}+5 m x_{2}+12 m}+\frac{x_{2}^{2}+5 m x_{1}+12 m}{m^{2}}$

Lời giải.

a) Định $m$ để phương trình có hai nghiệm phân biệt

ĐKXĐ đề phương trình có hai nghiệm phân biệt là:

$\Delta=(-5 m)^{2}-4(-4 m)=25 m^{2}+16 m=m(25 m+16)>0$

$\Leftrightarrow\left\{\begin{array}{l}m>0 \\ m<\frac{-16}{25}\end{array}\right.$

b) Tìm $m$ để biếu thức sau đạt giá trị nhỏ nhất:

$P=\frac{m^{2}}{x_{1}^{2}+5 m x_{2}+12 m}+\frac{x_{2}^{2}+5 m x_{1}+12 m}{m^{2}}$

Do $x_{1}, x_{2}$ là hai nghiệm của phương trình nên ta có: $\left\{\begin{array}{l}x_{1}^{2}=5 m x_{1}+4 m \\ x_{2}^{2}=5 m x_{2}+4 m\end{array}\right.$

Do phương trình đã cho có hai nghiệm phân biệt nên: $25 m^{2}+16 m>0$. Từ đó áp dụng bất đẳng thức Cauchy, ta có:

$P=\frac{m^{2}}{x_{1}^{2}+5 m x_{2}+12 m}+\frac{x_{2}^{2}+5 m x_{1}+12 m}{m^{2}}$

$P=\frac{m^{2}}{25 m^{2}+16 m}+\frac{25 m^{2}+16 m}{m^{2}} \geq 2$

Đẳng thức xảy ra khi và chỉ khi: $m^{2}=25 m^{2}+16 m \Leftrightarrow m=\frac{-2}{3}$

Bài 3. Cho tam giác $\triangle A B C$ có $B C$ là cạnh dài nhất. Trên $B C$ lấy hai điểm $D$ và $E$ sao cho $B D=B A, C E=C A$. Đường thẳng qua $D$ song song với $A B$ cắt $A C$ tại $M$. Đường thẳng qua $E$ song song với $A C$ cắt $A B$ tại $N$. Chứng minh rằng $A M=A N$.

Lời giải.

Do $D M | A B$, áp dụng định lí Talet:

$\frac{A M}{A C}=\frac{B D}{B C} \Leftrightarrow A M=\frac{B D}{B C} \cdot A C=\frac{B A \cdot A C}{B C}$

Do $E N | A C$, áp dụng định lí Talet:

$\frac{A N}{A B}=\frac{C E}{B C} \Leftrightarrow A N=\frac{C E}{B C} \cdot A B=\frac{B A \cdot A C}{B C}$

Từ đó ta có $A M=A N$. Đây chính là điều phải chứng minh.

Bài 4. Cho $x, y$ là hai số dương thỏa mãn: $x+y=1$. Chứng minh: $3(3 x-2)^{2}+\frac{8 x}{y} \geq 7$.

Lời giải. Do $x+y=1$ nên ta có điều phải chứng minh trở thành:

$3(3 x-2)^{2}+\frac{8 x}{1-x} \geq 7$

Bằng khai triển và biến đổi tương đương ta có: $(5-3 x)(3 x-1)^{2} \geq 0$. Bất đẳng thức này hiển nhiên đúng do $x<1$

Bài 5.Từ một điểm $A$ bên ngoài đường tròn $(O)$ vẽ các tiếp tuyến $A B, A C$ và cát tuyến $A E F$ ( $E F$ không đi qua $O, B$ và $C$ là các tiếp điểm). Gọi $D$ là điểm đối xứng của $B$ qua $O$. $D E, D F$ lần lượt cắt $A O$ tại $M$ và $N$. Chứng minh rằng :

a) Hai tam giác $\triangle C E F$ và $\triangle C M N$ đồng dạng.

b) $O M=O N$.

Lời giải.
a) Chứng minh rằng $\triangle C E F \backsim \triangle C M N$
Ta có: $A N | C D$ (cùng vuông góc với $B C$ )
$\angle D F C=\angle D B C=\angle B A O=\angle C A O$
Từ đó ta có: tứ giác $C F N A$ nội tiếp
Vậy: $\angle C F E=\angle C N M$
Ta có: $A N | C D$ nên: $\angle O M E=\angle C D E$
Do tứ giác $C D F E$ nội tiếp nên: $\angle C D E=\angle C F E$
Vậy: $\angle O M E=\angle C F E$
Mà: $\angle A C E=\angle C F E$ (Tính chất tiếp tuyến)
Từ đó ta có: $\angle A C E=\angle O M E$. Vậy tứ giác $A M E C$ nội tiếp. Nên: $\angle E A M=$ $\angle E C M$

Mà: $\angle E A M=\angle F C N$ (Tứ giác $A N F C$ nội tiếp)

Vậy: $\angle E C M=\angle F C N$

Từ đó ta có: $\angle E C F=\angle M C N$

Do: $\angle C F E=\angle C N M$ và $\angle E C F=\angle M C N$ nên ta có: $\triangle C E F \sim \triangle C M N$

b) Chứng minh rằng: $O M=O N$

Từ giác $A M E C$ nội tiếp: $\angle D C M=\angle C A F$

Từ giác $C F N A$ nội tiếp: $\angle C A F=\angle C N D$

Vậy ta có: $\angle D C M=\angle C N D$ và do: $A N | C D$. Vậy $C D N M$ là hình thang cân nên: $C N=D M$ và $\angle C N M=\angle D M N$

Do $A O$ là đường trung trực của $B C$ nên ta có: $\angle C N M=\angle B N M$ và $N C=N B$

Từ đó ta có: $\angle D M N=\angle B N M$ và $D M=B N$

Hay: $D M | B N$ và $D M=B N$. Từ đó $B M D N$ là hình bình hành. Mà $O$ là trung điểm của $B D$ nên $O$ cũng là trung điểm của $M N$ hay: $O M=O N$ (đpcm)

Bài 6. Chữ số hàng đơn vị trong hệ thập phân của số $M=a^{2}+a b+b^{2}$ là $0\left(a ; b \in N^{*}\right)$.

a) Chứng minh rằng $M$ chia hết cho 20 .

b) Tìm chữ số hàng chục của $M$.

Lới giải.

a) Chứng minh rằng: $M \vdots 20$

Do chữ số hàng đơn vị của $M$ là 0 nên ta có: $M \vdots 5$ và $M \vdots 2$

Giả sử cả $a$ và $b$ đều không chia hết cho 2 . Từ đó ta có:

$\left\{\begin{array} { l }{ a \equiv 1 } \\ { b \equiv 1 }\end{array} \Rightarrow \left\{\begin{array}{l}a^{2} \equiv 1 \\ b^{2} \equiv 1 \\ a b \equiv 1\end{array} \Rightarrow a^{2}+a b+b^{2} \equiv 1 \Rightarrow M \equiv 1(\bmod 2)\right.\right.$

Điều này vô lí: từ đó ta có trong hai số $a$ và $b$ phải có một số chia hết cho 2 .

Giả sử $a \vdots$ 2. Do $M \vdots 2$ nên $b^{2} \vdots 2$. Từ đó ta có: $b \vdots 2$

Vi $a \vdots 2$ và $b \vdots 2$ nên $M \vdots 4$

Do $M \vdots 4$ và $M \vdots 5$ nên ta có: $M \vdots 20$ (đpcm)

b) Nhận xét: Một số chính phương khi chia cho 5 dư 0,1 hoặc 4 .

Ta có $5 \mid a^{2}+a b+b^{2}$, suy ra $5 \mid 4 a^{2}+4 a b+4 b^{2}$ hay $5 \mid(2 a+b)^{2}+3 b^{2}$.

Từ nhận xét trên suy ra $5|b, 5| 2 a+b \Rightarrow 5 \mid a$. Do đó $a^{2}+a b+b^{2}$ chia hết cho $25 .$

Kết hợp với câu a ta có $M$ chia hết cho 100 nên chữ số hàng chục là số 0 .

 

 

 

 

 

 

 

 

 

 

 

 

 

 

ĐỀ THI VÀO LỚP 10 CHUYÊN TOÁN TP.HCM – NĂM 2014

Bài 1. (a) Giải phương trình: $x \sqrt{2 x-3}=3 x-4$

(b) Cho 3 số thực $x, y, z$ thỏa mãn điều kiện: $x+y+z=0 ; x y z \neq 0$. Tính giá trị biểu thức:

$P=\frac{x^{2}}{y^{2}+z^{2}-x^{2}}+\frac{y^{2}}{z^{2}+x^{2}-y^{2}}+\frac{z^{2}}{x^{2}+y^{2}-z^{2}}$

Bài 2. Giải hệ phương trình: $\left\{\begin{array}{l}x+y+\frac{1}{y}=\frac{9}{x} \\ x+y-\frac{4}{x}=\frac{4 y}{x^{2}}\end{array}\right.$

Bài 3. Cho tam giác đều $A B C$ và $M$ là một điểm bất kì trên cạnh $B C$. Gọi $D, E$ lần lượt là hình chiếu vuông góc của $M$ trên $A B$ và $A C$. Xác định vị trí của $M$ để tam giác $M D E$ có chu vi nhỏ nhất.

Bài 4. (a) Cho $x, y$ là 2 số thực khác 0 . Chứng minh rằng: $\frac{x^{2}}{y^{2}}+\frac{y^{2}}{x^{2}} \geq \frac{x}{y}+\frac{y}{x}$

(b) Cho $a, b$ là hai số dương. Tìm giá trị nhỏ nhất của biểu thức: $P=\frac{a^{2}+3 a b+b^{2}}{\sqrt{a b}(a+b)}$

Bài 5. Từ một điểm $M$ nằm ngoài đường tròn $(\mathrm{O})$, kẻ các tiếp tuyến $M A, M B$ với $(\mathrm{O})$ $(A, B$ là các tiếp điểm $)$. Gọi $H$ là giao điểm của $A B$ với $O M, I$ là trung điểm của $M H$. Đường thẳng $A I$ cắt $(\mathrm{O})$ tại điểm $K(K$ khác $A)$.

(a) Chứng minh $H K$ vuông góc với $A I$.

(b) Tính số đo góc $\angle M K B$.

Bài 6. Tìm cặp số nguyên $(x, y)$ thỏa mãn phương trình:

$2015\left(x^{2}+y^{2}\right)-2014(2 x y+1)=25$

LỜI GIẢI

 

Bài 1.

a) Giải phương trình: $x \sqrt{2 x-3}=3 x-4$

b) Cho 3 số thực $x, y, z$ thỏa mãn điều kiện: $x+y+z=0 ; x y z \neq 0$. Tính giá trị biểu thức:

$P=\frac{x^{2}}{y^{2}+z^{2}-x^{2}}+\frac{y^{2}}{z^{2}+x^{2}-y^{2}}+\frac{z^{2}}{x^{2}+y^{2}-z^{2}}$

Lời giải.

a) Giải phương trình: $x \sqrt{2 x-3}=3 x-4 Đ \mathrm{~K} Đ: x \geq \frac{3}{2}$

Phương trình đã cho tương đương với:

$x^{2}(2 x-3)=9 x^{2}-24 x+16 \Leftrightarrow 2 x^{3}-12 x^{2}+24 x-16=0 $

$\Leftrightarrow x^{3}-6 x^{2}+12 x-8=0 \Leftrightarrow(x-2)^{3}=0 \Leftrightarrow x=2$

Ta thấy $x=2$ thỏa yêu cầu bài toán, vậy $x=2$ là nghiệm duy nhất của phương trình.

b) Cho 3 số thực $x, y, z$ thỏa mãn điều kiện: $x+y+z=0 ; x y z \neq 0$. Tính giá trị biểu thức:

$P=\frac{x^{2}}{y^{2}+z^{2}-x^{2}}+\frac{y^{2}}{z^{2}+x^{2}-y^{2}}+\frac{z^{2}}{x^{2}+y^{2}-z^{2}}$

Ta có:

$y+z=-x \Leftrightarrow y^{2}+2 y z+z^{2}=x^{2} \Leftrightarrow y^{2}+z^{2}-x^{2}=-2 y z $

$x+z=-y \Leftrightarrow x^{2}+2 x z+z^{2}=y^{2} \Leftrightarrow x^{2}+z^{2}-y^{2}=-2 x z $

$y+x=-z \Leftrightarrow y^{2}+2 y x+x^{2}=z^{2} \Leftrightarrow y^{2}+x^{2}-z^{2}=-2 y x$

Từ đó ta tính được $P$ :

$P=\frac{x^{2}}{-2 y z}+\frac{y^{2}}{-2 x z}+\frac{z^{2}}{-2 y x}=\frac{x^{3}+y^{3}+z^{3}}{-2 x y z}$

Chú ý:

$x^{3}+y^{3}+z^{3}-3 x y z=0 \Rightarrow x^{3}+y^{3}+z^{3}=3 x y z$

Vậy: $P=\frac{x^{3}+y^{3}+z^{3}}{-2 x y z}=\frac{3 x y z}{-2 x y z}=\frac{-3}{2}$

Bài 2. Giải hệ phương trình: $\left\{\begin{array}{l}x+y+\frac{1}{y}=\frac{9}{x} \\ x+y-\frac{4}{x}=\frac{4 y}{x^{2}}\end{array}\right.$

Lời giải. ĐKXĐ: $x, y \neq 0$

Lấy phương trình (1) trừ phương trình (2) ta thu được:

$\frac{1}{y}+\frac{4}{x}=\frac{9}{x}-\frac{4 y}{x^{2}}  \Leftrightarrow \frac{1}{y}=\frac{5}{x}-\frac{4 y}{x^{2}} \Leftrightarrow x^{2}=5 x y-4 y^{2} \Leftrightarrow x^{2}-5 x y+4 y^{2}=0 $

$\Leftrightarrow(x-4 y)(x-y)=0 \Leftrightarrow\left[\begin{array}{l}x=4 y \\ x=y\end{array}\right.$

Trường hợp 1: $x=4 y$. Thay vào phương trình (1) ta có:

$5 y+\frac{1}{y}=\frac{9}{4 y} \Leftrightarrow 5 y=\frac{5}{4 y} \Leftrightarrow\left[\begin{array} { l }{ y = \frac { 1 } { 2 } } \\ { y = \frac { – 1 } { 2 } }\end{array} \Leftrightarrow \left[\begin{array}{l}x=2, y=\frac{1}{2} \\ x=-2, y=\frac{-1}{2}\end{array}\right.\right.$

Trường hợp $2: x=y$. Thay vào phương trình (1) ta có:

$2 y+\frac{1}{y}=\frac{9}{y} \Leftrightarrow 2 y=\frac{8}{y} \Leftrightarrow\left[\begin{array}{l}y=2 \\ y=-2\end{array} \Leftrightarrow\left[\begin{array}{l}x=2, y=2 \\ x=-2, y=-2\end{array}\right.\right.$

Vậy tập nghiệm của phương trình là: $(x, y)=(2,2),(-2,-2),\left(2, \frac{1}{2}\right),\left(-2, \frac{-1}{2}\right)$

Bài 3. Cho tam giác đều $A B C$ và $M$ là một điểm bất kì trên cạnh $B C$. Gọi $D, E$ lần lượt là hình chiếu vuông góc của $M$ trên $A B$ và $A C$. Xác định vị trí của $M$ để tam giác MDE có chu vi nhỏ nhất.

Lời giải.

  • Gọi độ dài cạnh tam giác đều là $a$.

Ta có $M D \cdot A B+M E \cdot A C=2 S_{A M D}+2 S_{A M C}=2 S_{A B C}$. Hay $(M D+M E)=A H \cdot a$, suy ra $M D+M E=A H$ không đổi.

  • Ta có $D, E$ thuộc đường tròn đường kính $A M$. Vẽ đường kính $D F$, ta có $\angle D F E=$ $\angle D A E=60^{\circ}$.

Suy ra $D E=D F \sin D F E=A M \sin 60^{\circ}$.

$D E$ nhỏ nhất khi và chỉ khi $A M$ nhỏ nhất, khi và chỉ khi $M$ trùng với $H$ trung điểm $B C$.

  • Vậy chu vi tam giác $M D E$ nhỏ nhất khi và chỉ khi $M$ là trung điểm $B C$.

Bài 4.

a) Cho $x, y$ là 2 số thực khác 0 . Chứng minh rằng: $\frac{x^{2}}{y^{2}}+\frac{y^{2}}{x^{2}} \geq \frac{x}{y}+\frac{y}{x}$

b) Cho $a, b$ là hai số dương. Tìm giá trị nhỏ nhất của biểu thức:

$P=\frac{a^{2}+3 a b+b^{2}}{\sqrt{a b}(a+b)}$

Lời giải.

a) Bằng biến đổi tương đương ta có:

$\frac{x^{2}}{y^{2}}+\frac{y^{2}}{x^{2}}-\left(\frac{x}{y}+\frac{y}{x}\right) \geq 0 \Leftrightarrow \frac{(x-y)^{2}\left(\left(x+\frac{1}{2}\right)^{2}+\frac{3}{4} y^{2}\right)}{x^{2} y^{2}} \geq 0$

Bất đẳng thức cuối luôn đúng. Dấu bằng trong bất đẳng thức xảy ra khi $x=y$.

b) Cách 1: Với $a, b$ là hai số dương. Ta có:

$P=\frac{a^{2}+3 a b+b^{2}}{\sqrt{a b}(a+b)}=\frac{(a+b)^{2}+a b}{\sqrt{a b}(a+b)}=\frac{\frac{1}{4}(a+b)^{2}+a b+\frac{3}{4}(a+b)^{2}}{\sqrt{a b}(a+b)} $

$P=\frac{\frac{1}{4}(a+b)^{2}+a b}{\sqrt{a b}(a+b)}+\frac{\frac{3}{4}(a+b)}{\sqrt{a b}}$

Áp dụng bất đẳng thức Cauchy:

$P=\frac{\frac{1}{4}(a+b)^{2}+a b}{\sqrt{a b}(a+b)}+\frac{\frac{3}{4}(a+b)}{\sqrt{a b}} \geq \frac{2 \sqrt{\frac{1}{4} a b(a+b)^{2}}}{\sqrt{a b}(a+b)}+\frac{\frac{3}{4} \cdot 2 \sqrt{a b}}{\sqrt{a b}}=1+\frac{3}{2}=\frac{5}{2}$

Dấu bằng trong bất đẳng thức xảy ra khi $a=b$

Cách 2: Ta có:

$P=\frac{a^{2}+3 a b+b^{2}}{\sqrt{a b}(a+b)}=\frac{(a+b)^{2}+a b}{\sqrt{a b}(a+b)}=\frac{a+b}{\sqrt{a b}}+\frac{\sqrt{a b}}{a+b}=\frac{3}{4} \cdot \frac{a+b}{\sqrt{a b}}+\frac{1}{4} \cdot \frac{a+b}{\sqrt{a b}}+\frac{\sqrt{a b}}{a+b}$

Áp dụng bất đẳng thức Cauchy:

$P \geq \frac{3}{4} \cdot 2+2 \sqrt{\frac{1}{4} \cdot \frac{a+b}{\sqrt{a b}} \cdot \frac{\sqrt{a b}}{a+b}}=\frac{3}{2}+1=\frac{5}{2}$

Dấu bằng trong bất đẳng thức xảy ra khi $a=b$

Bài 5.Từ một điểm $M$ nằm ngoài đường tròn $(\mathrm{O})$, kẻ các tiếp tuyến $M A, M B$ với $(\mathrm{O})$ $(A, B$ là các tiếp điểm $)$. Gọi $H$ là giao điểm của $A B$ với $O M, I$ là trung điểm của $M H$. Đường thẳng $A I$ cắt $(\mathrm{O})$ tại điểm $K(K$ khác $A)$.

a) Chứng minh $H K$ vuông góc với $A I$.

b) Tính số đo góc $\angle M K B$.

Lời giải.

a) Vẽ đường kính $A C, C H$ cắt $A I$ tại $K^{\prime}$.

Dễ thấy hai tam giác $A B C$ và $M H A$ đồng dạng, từ đó suy ra $A C H$ và $M A I$ đồng dạng.

Suy ra $\angle A C H=\angle M A I$, mà $\angle M A I+\angle I A C=90^{\circ}$, suy ra $\angle A C H+\angle I A C=$ $90^{\circ}$.

Do đó $\angle A K^{\prime} C=90^{\circ}$, suy ra $K^{\prime}$ thuộc $(O)$, từ đó $K^{\prime} \equiv K$. Ta có điều cần chứng minh.

b) Ta có $I K \cdot I A=I H^{2}=I M^{2}$.

Suy ra $\triangle I K M \backsim \triangle I M A$, do đó $\angle I M K=\angle I A M=\angle K B H$.

Từ đó tứ giác $B H K M$ nội tiếp, suy ra $\angle B K M=\angle B H M=90^{\circ}$.

Bài 6. Tìm cặp số nguyên $(x, y)$ thỏa mãn phương trình:

$2015\left(x^{2}+y^{2}\right)-2014(2 x y+1)=25$

Lời giải.

Ta có: $2015\left(x^{2}+y^{2}\right)-2014(2 x y+1)=25$

$\Leftrightarrow 2014(x-y)^{2}+x^{2}+y^{2}=2039$

Vậy: $2014(x-y)^{2} \leq 2039 \Leftrightarrow|x-y| \leq 1$

  • Trường hợp 1: $x-y=0$. Ta có: $x^{2}+y^{2}=2039$

Phương trình này không có nghiệm nguyên vì 2039 không chia hết cho $2 .$

  • Trường hợp 2: $x-y=1$. Ta có: $y^{2}+y-12=0$

Phương trình này có nghiệm $y=3$ hay $y=-4$

Từ đó ta có hai cặp nghiệm của phương trình là: $(x, y)={(4 ; 3),(-3 ;-4)}$

  • Trường hợp 3: $x-y=-1$. Ta có: $y^{2}-y-12=0$

Phương trình này có nghiệm $y=-3$ hay $y=4$

Từ đó ta có hai cặp nghiệm của phương trình là: $(x, y)={(-4 ;-3),(3 ; 4)}$

Vậy tập nghiệm của phương trình là: $(x, y)={(4 ; 3),(-3 ;-4),(3 ; 4),(-4 ;-3)}$